maternity 2

Ace your homework & exams now with Quizwiz!

Before the physician performs an external version, the nurse should expect an order for a a. tocolytic drug. b. contraction stress test (CST). c. local anesthetic. d. Foley catheter

ANS: A A tocolytic drug will relax the uterus before and during version, thus making manipulation easier. CST is used to determine the fetal response to stress. A local anesthetic is not used with external version. The bladder should be emptied; however, catheterization is not necessary.

As the United States and Canada continue to become more culturally diverse, it is increasingly important for the nursing staff to recognize a wide range of varying cultural beliefs and practices. Nurses need to develop respect for these culturally diverse practices and learn to incorporate these into a mutually agreed on plan of care. Although it is common practice in the United States for the father of the baby to be present at the birth, in many societies this is not the case. When implementing care, the nurse would anticipate that a woman from which country would have the father of the baby in attendance? a. Mexico b. China c. Iran d. India

ANS: A A woman from Mexico may be stoic about discomfort until the second stage, at which time she will request pain relief. Fathers and female relatives are usually in attendance during the second stage of labor. The father of the baby is expected to provide encouragement, support, and reassurance that all will be well. Fathers are usually not present in China. The Iranian father will not be present. Female support persons and female care providers are preferred. For many, a male caregiver is unacceptable. The father is usually not present in India, but female relatives are usually present. Natural childbirth methods are preferred

Nursing care measures are commonly offered to women in labor. Which nursing measure reflects application of the gate-control theory? a. Massaging the woman's back b. Changing the woman's position c. Giving the prescribed medication d. Encouraging the woman to rest between contractions

ANS: A According to the gate-control theory, pain sensations travel along sensory nerve pathways to the brain, but only a limited number of sensations, or messages, can travel through these nerve pathways at one time. Distraction techniques such as massage or stroking, music, focal points, and imagery reduce or completely block the capacity of nerve pathways to transmit pain. These distractions are thought to work by closing down a hypothetic gate in the spinal cord and thus preventing pain signals from reaching the brain. The perception of pain is thereby diminished. Changing the woman's position, giving prescribed medication, and encouraging rest do not reduce or block the capacity of nerve pathways to transmit pain using the gate-control theory.

The nurse providing newborn stabilization must be aware that the primary side effect of maternal narcotic analgesia in the newborn is a. respiratory depression. b. bradycardia. c. acrocyanosis. d. tachypnea

ANS: A An infant delivered within 1 to 4 hours of maternal analgesic administration is at risk for respiratory depression from the sedative effects of the narcotic. Bradycardia is not the anticipated side effect of maternal analgesics. Acrocyanosis is an expected finding in a newborn and is not related to maternal analgesics. The infant who is having a side effect to maternal analgesics normally would have a decrease in respirations, not an increase

A woman in preterm labor at 30 weeks of gestation receives two 12-mg doses of betamethasone intramuscularly. The purpose of this pharmacologic treatment is to a. stimulate fetal surfactant production. b. reduce maternal and fetal tachycardia associated with ritodrine administration. c. suppress uterine contractions. d. maintain adequate maternal respiratory effort and ventilation during magnesium sulfate therapy.

ANS: A Antenatal glucocorticoids given as intramuscular injections to the mother accelerate fetal lung maturity. Inderal would be given to reduce the effects of ritodrine administration. Betamethasone has no effect on uterine contractions. Calcium gluconate would be given to reverse the respiratory depressive effects of magnesium sulfate therapy

Which occurrence is associated with cervical dilation and effacement? a. Bloody show b. False labor c. Lightening d. Bladder distention

ANS: A As the cervix begins to soften, dilate, and efface, expulsion of the mucous plug that sealed the cervix during pregnancy occurs. This causes rupture of small cervical capillaries. Cervical dilation and effacement do not occur with false labor. Lightening is the descent of the fetus toward the pelvic inlet before labor. Bladder distention occurs when the bladder is not emptied frequently. It may slow down the descent of the fetus during labor.

With regard to the process of augmentation of labor, the nurse should be aware that it a. is part of the active management of labor that is instituted when the labor process is unsatisfactory. b. relies on more invasive methods when oxytocin and amniotomy have failed. c. is a modern management term to cover up the negative connotations of forceps-assisted birth. d. uses vacuum cups.

ANS: A Augmentation is part of the active management of labor that stimulates uterine contractions after labor has started but is not progressing satisfactorily. Augmentation uses amniotomy and oxytocin infusion, as well as some gentler, noninvasive methods. Forceps-assisted births and vacuum-assisted births are appropriately used at the end of labor and are not part of augmentation

With regard to nerve block analgesia and anesthesia, nurses should be aware that a. most local agents are related chemically to cocaine and end in the suffix -caine. b. local perineal infiltration anesthesia is effective when epinephrine is added, but it can be injected only once. c. a pudendal nerve block is designed to relieve the pain from uterine contractions. d. a pudendal nerve block, if done correctly, does not significantly lessen the bearing-down reflex.

ANS: A Common agents include lidocaine and chloroprocaine. Injections can be repeated to prolong the anesthesia. A pudendal nerve block relieves pain in the vagina, vulva, and perineum but not the pain from uterine contractions, and it lessens or shuts down the bearing-down reflex.

A normal uterine activity pattern in labor is characterized by a. contractions every 2 to 5 minutes. b. contractions lasting about 2 minutes. c. contractions about 1 minute apart. d. a contraction intensity of about 1000 mm Hg with relaxation at 50 mm Hg

ANS: A Contractions normally occur every 2 to 5 minutes and last less than 90 seconds (intensity 800 mm Hg) with about 30 seconds in between (20 mm Hg or less)

A woman is experiencing back labor and complains of intense pain in her lower back. An effective relief measure would be to use a. counterpressure against the sacrum. b. pant-blow (breaths and puffs) breathing techniques. c. effleurage. d. conscious relaxation or guided imagery.

ANS: A Counterpressure is a steady pressure applied by a support person to the sacral area with the fist or heel of the hand. This technique helps the woman cope with the sensations of internal pressure and pain in the lower back. The pain-management techniques of pant-blow, effleurage, and conscious relaxation or guided imagery are usually helpful for contractions per the gate-control theory

The nurse would expect which maternal cardiovascular finding during labor? a. Increased cardiac output b. Decreased pulse rate c. Decreased white blood cell (WBC) count d. Decreased blood pressure

ANS: A During each contraction, 400 mL of blood is emptied from the uterus into the maternal vascular system. This increases cardiac output by about 51% above baseline pregnancy values at term. The heart rate increases slightly during labor. The WBC count can increase during labor. During the first stage of labor, uterine contractions cause systolic readings to increase by about 10 mm Hg. During the second stage, contractions may cause systolic pressures to increase by 30 mm Hg and diastolic readings to increase by 25 mm Hg

Which deceleration of the fetal heart rate would not require the nurse to change the maternal position? a. Early decelerations b. Late decelerations c. Variable decelerations d. It is always a good idea to change the woman's position

ANS: A Early decelerations (and accelerations) generally do not need any nursing intervention. Late decelerations suggest that the nurse should change the maternal position (lateral); variable decelerations also require a maternal position change (side to side). Although changing positions throughout labor is recommended, it is not required in response to early decelerations.

The nurse caring for the laboring woman should understand that early decelerations are caused by a. altered fetal cerebral blood flow. b. umbilical cord compression. c. uteroplacental insufficiency. d. spontaneous rupture of membranes.

ANS: A Early decelerations are the fetus's response to fetal head compression. Variable decelerations are associated with umbilical cord compression. Late decelerations are associated with uteroplacental insufficiency. Spontaneous rupture of membranes has no bearing on the fetal heart rate unless the umbilical cord prolapses, which would result in variable or prolonged bradycardia

The nurse providing care for the laboring woman understands that accelerations with fetal movement a. are reassuring. b. are caused by umbilical cord compression. c. warrant close observation. d. are caused by uteroplacental insufficiency.

ANS: A Episodic accelerations in the fetal heart rate (FHR) occur during fetal movement and are indications of fetal well-being. Umbilical cord compression results in variable decelerations in the FHR. Accelerations in the FHR are an indication of fetal well-being and do not warrant close observation. Uteroplacental insufficiency would result in late decelerations in the FHR.

Fetal well-being during labor is assessed by a. the response of the fetal heart rate (FHR) to uterine contractions (UCs). b. maternal pain control. c. accelerations in the FHR. d. an FHR above 110 beats/min

ANS: A Fetal well-being during labor can be measured by the response of the FHR to UCs. In general, reassuring FHR patterns are characterized by an FHR baseline in the range of 110 to 160 beats/min with no periodic changes, a moderate baseline variability, and accelerations with fetal movement. Maternal pain control is not the measure used to determine fetal well-being in labor. Although FHR accelerations are a reassuring pattern, they are only one component of the criteria by which fetal well-being is assessed. Although an FHR above 110 beats/min may be reassuring, it is only one component of the criteria by which fetal well-being is assessed. More information would be needed to determine fetal well-being

With regard to breathing techniques during labor, maternity nurses should understand that a. breathing techniques in the first stage of labor are designed to increase the size of the abdominal cavity to reduce friction. b. by the time labor has begun, it is too late for instruction in breathing and relaxation. c. controlled breathing techniques are most difficult near the end of the second stage of labor. d. the patterned-paced breathing technique can help prevent hyperventilation.

ANS: A First-stage techniques promote relaxation of abdominal muscles, thereby increasing the size of the abdominal cavity. Instruction in simple breathing and relaxation techniques early in labor is possible and effective. Controlled breathing techniques are most difficult in the transition phase at the end of the first stage of labor when the cervix is dilated 8 to 10 cm. Patterned-paced breathing sometimes can lead to hyperventilation

Which description of the four stages of labor is correct for both definition and duration? a. First stage: onset of regular uterine contractions to full dilation; less than 1 hour to 20 hours b. Second stage: full effacement to 4 to 5 cm; visible presenting part; 1 to 2 hours c. Third state: active pushing to birth; 20 minutes (multiparous women), 50 minutes (first-timer) d. Fourth stage: delivery of the placenta to recovery; 30 minutes to 1 hour

ANS: A Full dilation may occur in less than 1 hour, but in first-time pregnancy it can take up to 20 hours. The second stage extends from full dilation to birth and takes an average of 20 to 50 minutes, although 2 hours is still considered normal. The third stage extends from birth to expulsion of the placenta and usually takes a few minutes. The fourth stage begins after expulsion of the placenta and lasts until homeostasis is reestablished (about 2 hours).

Which presentation is described accurately in terms of both presenting part and frequency of occurrence? a. Cephalic: occiput; at least 95% b. Breech: sacrum; 10% to 15% c. Shoulder: scapula; 10% to 15% d. Cephalic: cranial; 80% to 85%

ANS: A In cephalic presentations (head first), the presenting part is the occiput; this occurs in 96% of births. In a breech birth, the sacrum emerges first; this occurs in about 3% of births. In shoulder presentations, the scapula emerges first; this occurs in only 1% of births.

Which maternal condition is considered a contraindication for the application of internal monitoring devices? a. Unruptured membranes b. Cervix dilated to 4 cm c. External monitors in current use d. Fetus with a known heart defect

ANS: A In order to apply internal monitoring devices, the membranes must be ruptured. Cervical dilation of 4 cm permits the insertion of fetal scalp electrodes and intrauterine catheter. The external monitor can be discontinued after the internal ones are applied. A compromised fetus should be monitored with the most accurate monitoring devices

Leopold maneuvers would be an inappropriate method of assessment to determine a. gender of the fetus. b. number of fetuses. c. fetal lie and attitude. d. degree of the presenting part's descent into the pelvis.

ANS: A Leopold maneuvers help identify the number of fetuses, the fetal lie and attitude, and the degree of descent of the presenting part into the pelvis. The gender of the fetus is not a goal of the examination at this time.

The nurse caring for a laboring woman is aware that maternal cardiac output can be increased by a. change in position. b. oxytocin administration. c. regional anesthesia. d. intravenous analgesic

ANS: A Maternal supine hypotension syndrome is caused by the weight and pressure of the gravid uterus on the ascending vena cava when the woman is in a supine position. This reduces venous return to the woman's heart, as well as cardiac output, and subsequently reduces her blood pressure. The nurse can encourage the woman to change positions and avoid the supine position. Oxytocin administration, regional anesthesia, and intravenous analgesic may reduce maternal cardiac output.

Perinatal nurses are legally responsible for a. correctly interpreting fetal heart rate (FHR) patterns, initiating appropriate nursing interventions, and documenting the outcomes. b. greeting the patient on arrival, assessing her, and starting an intravenous line. c. applying the external fetal monitor and notifying the care provider. d. making sure that the woman is comfortable.

ANS: A Nurses who care for women during childbirth are legally responsible for correctly interpreting FHR patterns, initiating appropriate nursing interventions based on those patterns, and documenting the outcomes of those interventions. Greeting the patient, assessing her, and starting an IV; applying the external fetal monitor and notifying the care provider; and making sure the woman is comfortable may be activities that a nurse performs, but they are not activities for which the nurse is legally responsible.

When assessing a multiparous woman who has just given birth to an 8-lb boy, the nurse notes that the woman's fundus is firm and has become globular in shape. A gush of dark red blood comes from her vagina. The nurse concludes that a. the placenta has separated. b. a cervical tear occurred during the birth. c. the woman is beginning to hemorrhage. d. clots have formed in the upper uterine segment

ANS: A Placental separation is indicated by a firmly contracting uterus, a change in the uterus from a discoid to a globular ovoid shape, a sudden gush of dark red blood from the introitus, an apparent lengthening of the umbilical cord, and a finding of vaginal fullness. Cervical tears that do not extend to the vagina result in minimal blood loss. Signs of hemorrhage are a boggy uterus, bright red vaginal bleeding, alterations in vital signs, pallor, lightheadedness, restlessness, decreased urinary output, and alteration in the level of consciousness. If clots have formed in the upper uterine segment, the nurse would expect to find the uterus boggy and displaced to the side.

Nurses should be aware of the differences experience can make in labor pain such as a. sensory pain for nulliparous women often is greater than for multiparous women during early labor. b. affective pain for nulliparous women usually is less than for multiparous women throughout the first stage of labor. c. women with a history of substance abuse experience more pain during labor. d. multiparous women have more fatigue from labor and therefore experience more pain.

ANS: A Sensory pain is greater for nulliparous women because their reproductive tract structures are less supple. Affective pain is higher for nulliparous women during the first stage but decreases for both nulliparous and multiparous during the second stage. Women with a history of substance abuse experience the same amount of pain as those without such a history. Nulliparous women have longer labors and therefore experience more fatigue.

A means of controlling the birth of the fetal head with a vertex presentation is a. the Ritgen maneuver. b. fundal pressure. c. the lithotomy position. d. the De Lee apparatus

ANS: A The Ritgen maneuver extends the head during the actual birth and protects the perineum. Gentle, steady pressure against the fundus of the uterus facilitates vaginal birth. The lithotomy position has been commonly used in Western cultures, partly because it is convenient for the health care provider. The De Lee apparatus is used to suction fluid from the infant's mouth.

When using intermittent auscultation (IA) to assess uterine activity, the nurse should be cognizant that a. the examiner's hand should be placed over the fundus before, during, and after contractions. b. the frequency and duration of contractions is measured in seconds for consistency. c. contraction intensity is given a judgment number of 1 to 7 by the nurse and patient together. d. the resting tone between contractions is described as either placid or turbulent.

ANS: A The assessment is done by palpation; duration, frequency, intensity, and resting tone must be assessed. The duration of contractions is measured in seconds; the frequency is measured in minutes. The intensity of contractions usually is described as mild, moderate, or strong. The resting tone usually is characterized as soft or relaxed.

The most critical nursing action in caring for the newborn immediately after birth is a. keeping the newborn's airway clear. b. fostering parent-newborn attachment. c. drying the newborn and wrapping the infant in a blanket. d. administering eyedrops and vitamin K

ANS: A The care given immediately after the birth focuses on assessing and stabilizing the newborn. Although fostering parent-infant attachment is an important task for the nurse, it is not the most critical nursing action in caring for the newborn immediately after birth. The nursing activities would be (in order of importance) to maintain a patent airway, support respiratory effort, and prevent cold stress by drying the newborn and covering the infant with a warmed blanket or placing the newborn under a radiant warmer. After the newborn has been stabilized, the nurse assesses the newborn's physical condition, weighs and measures the newborn, administers prophylactic eye ointment and a vitamin K injection, affixes an identification bracelet, wraps the newborn in warm blankets, and then gives the infant to the partner or mother when he or she is ready.

To care for a laboring woman adequately, the nurse understands that the stage of labor varies the most in length? a. first b. second c. third d. fourth

ANS: A The first stage of labor is considered to last from the onset of regular uterine contractions to full dilation of the cervix. The first stage is much longer than the second and third stages combined. In a first-time pregnancy the first stage of labor can take up to 20 hours. The second stage of labor lasts from the time the cervix is fully dilated to the birth of the fetus. The average length is 20 minutes for a multiparous woman and 50 minutes for a nulliparous woman. The third stage of labor lasts from the birth of the fetus until the placenta is delivered. This stage may be as short as 3 minutes or as long as 1 hour. The fourth stage of labor, recovery, lasts about 2 hours after delivery of the placenta.

In evaluating the effectiveness of oxytocin induction, the nurse would expect a. contractions lasting 80 to 90 seconds, 2 to 3 minutes apart. b. the intensity of contractions to be at least 110 to 130 mm Hg. c. labor to progress at least 2 cm/hr dilation. d. at least 30 mU/min of oxytocin will be needed to achieve cervical dilation

ANS: A The goal of induction of labor would be to produce contractions that occur every 2 to 3 minutes and last 60 to 90 seconds. The intensity of the contractions should be 80 to 90 mm Hg by intrauterine pressure catheter. Cervical dilation of 1 cm/hr in the active phase of labor would be the goal in an oxytocin induction. The dose is increased by 1 to 2 mU/min at intervals of 30 to 60 minutes until the desired contraction pattern is achieved. Doses are increased up to a maximum of 20 to 40 mU/min

Immediately after the forceps-assisted birth of an infant, the nurse should a. assess the infant for signs of trauma. b. give the infant prophylactic antibiotics. c. apply a cold pack to the infant's scalp. d. measure the circumference of the infant's head.

ANS: A The infant should be assessed for bruising or abrasions at the site of application, facial palsy, and subdural hematoma. Prophylactic antibiotics are not necessary with a forceps delivery. A cold pack would put the infant at risk for cold stress and is contraindicated. Measuring the circumference of the head is part of the initial nursing assessment.

The exact cause of preterm labor is unknown and believed to be multifactorial. Infection is thought to be a major factor in many preterm labors. Select the type of infection that has not been linked to preterm births. a. Viral b. Periodontal c. Cervical d. Urinary tract

ANS: A The infections that increase the risk of preterm labor and birth are all bacterial. They include cervical, urinary tract, periodontal, and other bacterial infections. Therefore, it is important for the patient to participate in early, continual, and comprehensive prenatal care. Evidence has shown a link between periodontal infections and preterm labor. Researchers recommend regular dental care before and during pregnancy, oral assessment as a routine part of prenatal care, and scrupulous oral hygiene to prevent infection. Cervical infections of a bacterial nature have been linked to preterm labor and birth. The presence of urinary tract infections increases the risk of preterm labor and birth.

The primary difference between the labor of a nullipara and that of a multipara is the a. amount of cervical dilation. b. total duration of labor. c. level of pain experienced. d. sequence of labor mechanisms

ANS: B Multiparas usually labor more quickly than nulliparas, thus making the total duration of their labor shorter. Cervical dilation is the same for all labors. The level of pain is individual to the woman, not to the number of labors she has experienced. The sequence of labor mechanisms remains the same with all labors

In relation to primary and secondary powers, the maternity nurse comprehends that a. primary powers are responsible for effacement and dilation of the cervix. b. effacement generally is well ahead of dilation in women giving birth for the first time; they are closer together in subsequent pregnancies. c. scarring of the cervix caused by a previous infection or surgery may make the delivery a bit more painful, but it should not slow or inhibit dilation. d. pushing in the second stage of labor is more effective if the woman can breathe deeply and control some of her involuntary needs to push, as the nurse directs.

ANS: A The primary powers are responsible for dilation and effacement; secondary powers are concerned with expulsion of the fetus. Effacement generally is well ahead of dilation in first-timers; they are closer together in subsequent pregnancies. Scarring of the cervix may slow dilation. Pushing is more effective and less fatiguing when the woman begins to push only after she has the urge to do so.

The nurse knows that proper placement of the tocotransducer for electronic fetal monitoring is located a. over the uterine fundus. b. on the fetal scalp. c. inside the uterus. d. over the mother's lower abdomen.

ANS: A The tocotransducer monitors uterine activity and should be placed over the fundus, where the most intensive uterine contractions occur. The tocotransducer is for external use.

When assessing a woman in the first stage of labor, the nurse recognizes that the most conclusive sign that uterine contractions are effective would be a. dilation of the cervix. b. descent of the fetus. c. rupture of the amniotic membranes. d. increase in bloody show.

ANS: A The vaginal examination reveals whether the woman is in true labor. Cervical change, especially dilation, in the presence of adequate labor indicates that the woman is in true labor. Descent of the fetus, or engagement, may occur before labor. Rupture of membranes may occur with or without the presence of labor. Bloody show may indicate slow, progressive cervical change (e.g., effacement) in both true and false labor.

A pregnant woman's amniotic membranes rupture. Prolapsed umbilical cord is suspected. What intervention would be the top priority? a. Placing the woman in the knee-chest position b. Covering the cord in sterile gauze soaked in saline c. Preparing the woman for a cesarean birth d. Starting oxygen by face mask

ANS: A The woman is assisted into a position (e.g., modified Sims position, Trendelenburg position, or the knee-chest position) in which gravity keeps the pressure of the presenting part off the cord. Although covering the cord in sterile gauze-soaked saline, preparing the woman for a cesarean, and starting oxygen by face mark are appropriate nursing interventions in the event of a prolapsed cord, the intervention of top priority would be positioning the mother to relieve cord compression

In assisting with the two factors that have an effect on fetal status (i.e., pushing and positioning), nurses should a. encourage the woman's cooperation in avoiding the supine position. b. advise the woman to avoid the semi-Fowler position. c. encourage the woman to hold her breath and tighten her abdominal muscles to produce a vaginal response. d. instruct the woman to open her mouth and close her glottis, letting air escape after the push

ANS: A The woman should maintain a side-lying position. The semi-Fowler position is the recommended side-lying position with a lateral tilt to the uterus. The Valsalva maneuver, which encourages the woman to hold her breath and tighten her abdominal muscles, should be avoided. Both the mouth and glottis should be open, letting air escape during the push.

A laboring woman is lying in the supine position. The most appropriate nursing action at this time is to a. ask her to turn to one side. b. elevate her feet and legs. c. take her blood pressure. d. determine whether fetal tachycardia is present.

ANS: A The woman's supine position may cause the heavy uterus to compress her inferior vena cava, thus reducing blood return to her heart and reducing placental blood flow. Elevating her legs will not relieve the pressure from the inferior vena cava. If the woman is allowed to stay in the supine position and blood flow to the placental is reduced significantly, fetal tachycardia may occur. The most appropriate nursing action is to prevent this from occurring by turning the woman to her side. Blood pressure readings may be obtained when the patient is in the appropriate and safest position.

To help patients manage discomfort and pain during labor, nurses should be aware that a. the predominant pain of the first stage of labor is the visceral pain located in the lower portion of the abdomen. b. referred pain is the extreme discomfort between contractions. c. the somatic pain of the second stage of labor is more generalized and related to fatigue. d. pain during the third stage is a somewhat milder version of the second stage.

ANS: A This pain comes from cervical changes, distention of the lower uterine segment, and uterine ischemia. Referred pain occurs when the pain that originates in the uterus radiates to the abdominal wall, lumbosacral area of the back, iliac crests, and gluteal area. Second-stage labor pain is intense, sharp, burning, and localized. Third-stage labor pain is similar to that of the first stage.

A pregnant woman is in her third trimester. She asks the nurse to explain how she can tell true labor from false labor. The nurse would explain that ―true‖ labor contractions a. increase with activity such as ambulation. b. decrease with activity. c. are always accompanied by the rupture of the bag of waters. d. alternate between a regular and an irregular pattern.

ANS: A True labor contractions become more intense with walking. False labor contractions often stop with walking or position changes. Rupture of membranes may occur before or during labor. True labor contractions are regular

When managing the care of a woman in the second stage of labor, the nurse uses various measures to enhance the progress of fetal descent. These measures include a. encouraging the woman to try various upright positions, including squatting and standing. b. telling the woman to start pushing as soon as her cervix is fully dilated. c. continuing an epidural anesthetic so pain is reduced and the woman can relax. d. coaching the woman to use sustained, 10- to 15-second, closed-glottis bearing-down efforts with each contraction.

ANS: A Upright positions and squatting both may enhance the progress of fetal descent. Many factors dictate when a woman will begin pushing. Complete cervical dilation is necessary, but it is only one factor. If the fetal head is still in a higher pelvic station, the physician or midwife may allow the woman to ―labor down‖ (allowing more time for fetal descent, thereby reducing the amount of pushing needed) if the woman is able. The epidural may mask the sensations and muscle control needed for the woman to push effectively. Closed glottic breathing may trigger the Valsalva maneuver, which increases intrathoracic and cardiovascular pressures, reducing cardiac output and inhibiting perfusion of the uterus and placenta. In addition, holding the breath for longer than 5 to 7 seconds diminishes the perfusion of oxygen across the placenta and results in fetal hypoxia

When planning care for a laboring woman whose membranes have ruptured, the nurse recognizes that the woman's risk for has increased. a. intrauterine infection b. hemorrhage c. precipitous labor d. supine hypotension

ANS: A When the membranes rupture, microorganisms from the vagina can ascend into the amniotic sac and cause chorioamnionitis and placentitis. Rupture of membranes (ROM) is not associated with fetal or maternal bleeding. Although ROM may increase the intensity of contractions and facilitate active labor, it does not result in precipitous labor. ROM has no correlation with supine hypotension.

With regard to a woman's intake and output during labor, nurses should be aware that a. the tradition of restricting the laboring woman to clear liquids and ice chips is being challenged because regional anesthesia is used more often than general anesthesia. b. intravenous (IV) fluids usually are necessary to ensure that the laboring woman stays hydrated. c. routine use of an enema empties the rectum and is very helpful for producing a clean, clear delivery. d. when a nulliparous woman experiences the urge to defecate, it often means birth will follow quickly

ANS: A Women are awake with regional anesthesia and are able to protect their own airway, which reduces the worry over aspiration. Routine IV fluids during labor are unlikely to be beneficial and may be harmful. Routine use of an enema is at best ineffective and may be harmful. A multiparous woman may feel the urge to defecate and it may mean birth will follow quickly, but not for a first timer

While developing an intrapartum care plan for the patient in early labor, it is important that the nurse recognize that psychosocial factors may influence a woman's experience of pain. These include (Select all that apply.) a. culture. b. anxiety and fear. c. previous experiences with pain. d. intervention of caregivers. e. support systems.

ANS: A, B, C, E Culture: A woman's sociocultural roots influence how she perceives, interprets, and responds to pain during childbirth. Some cultures encourage loud and vigorous expressions of pain, whereas others value self-control. The nurse should avoid praising some behaviors (stoicism) while belittling others (noisy expression). Anxiety and fear: Extreme anxiety and fear magnify sensitivity to pain and impair a woman's ability to tolerate it. Anxiety and fear increase muscle tension in the pelvic area, which counters the expulsive forces of uterine contractions and pushing efforts. Previous experiences with pain: Fear and withdrawal are a natural response to pain during labor. Learning about these normal sensations ahead of time helps a woman suppress her natural reactions of fear regarding the impending birth. If a woman previously had a long and difficult labor, she is likely to be anxious. She may also have learned ways to cope and may use these skills to adapt to the present labor experience. Support systems: An anxious partner is less able to provide help and support to a woman during labor. A woman's family and friends can be an important source of support if they convey realistic and positive information about labor and delivery. Although the intervention of caregivers may be necessary for the well-being of the woman and her fetus, some interventions add discomfort to the natural pain of labor (i.e., fetal monitor straps, intravenous lines).

Signs that precede labor include (Select all that apply.) a. lightening. b. exhaustion. c. bloody show. d. rupture of membranes. e. decreased fetal movement.

ANS: A, C, D Signs that precede labor may include lightening, urinary frequency, backache, weight loss, surge of energy, bloody show, and rupture of membranes. Many women experience a burst of energy before labor. A decrease in fetal movement is an ominous sign that does not always correlate with labor.

The class of drugs known as opioid analgesics (butorphanol, nalbuphine) is not suitable for administration to women with known opioid dependence. The antagonistic activity could precipitate withdrawal symptoms (abstinence syndrome) in both mothers and newborns. Signs of opioid/narcotic withdrawal in the mother would include (Select all that apply.) a. yawning, runny nose. b. increase in appetite. c. chills and hot flashes. d. constipation. e. irritability, restlessness

ANS: A, C, E The woman experiencing maternal opioid withdrawal syndrome will exhibit yawning, runny nose, sneezing, anorexia, chills or hot flashes, vomiting, diarrhea, abdominal pain, irritability, restlessness, muscle spasms, weakness, and drowsiness. It is important for the nurse to assess both mother and baby and to plan care accordingly.

With regard to dysfunctional labor, nurses should be aware that a. women who are underweight are more at risk. b. women experiencing precipitous labor are about the only ―dysfunctionals‖ not to be exhausted. c. hypertonic uterine dysfunction is more common than hypotonic dysfunction. d. abnormal labor patterns are most common in older women.

ANS: B Precipitous labor lasts less than 3 hours. Short women more than 30 lbs overweight are more at risk for dysfunctional labor. Hypotonic uterine dysfunction, in which the contractions become weaker, is more common. Abnormal labor patterns are more common in women less than 20 years of age.

Complications and risks associated with cesarean births include (Select all that apply.) a. placental abruption. b. wound dehiscence. c. hemorrhage. d. urinary tract infections. e. fetal injuries.

ANS: B, C, D, E Placental abruption and placenta previa are both indications for cesarean birth and are not complications thereof. Wound dehiscence, hemorrhage, urinary tract infection, and fetal injuries are all possible complications and risks associated with delivery by cesarean section

The nerve block used in labor that provides anesthesia to the lower vagina and perineum is called a. an epidural. b. a pudendal. c. a local. d. a spinal block.

ANS: B A pudendal block anesthetizes the lower vagina and perineum to provide anesthesia for an episiotomy and use of low forceps if needed. An epidural provides anesthesia for the uterus, perineum, and legs. A local provides anesthesia for the perineum at the site of the episiotomy. A spinal block provides anesthesia for the uterus, perineum, and down the legs

Concerning the third stage of labor, nurses should be aware that a. the placenta eventually detaches itself from a flaccid uterus. b. an expectant or active approach to managing this stage of labor reduces the risk of complications. c. it is important that the dark, roughened maternal surface of the placenta appear before the shiny fetal surface. d. the major risk for women during the third stage is a rapid heart rate

ANS: B Active management facilitates placental separation and expulsion, thus reducing the risk of complications. The placenta cannot detach itself from a flaccid (relaxed) uterus. Which surface of the placenta comes out first is not clinically important. The major risk for women during the third stage of labor is after birth hemorrhage.

A woman who is gravida 3 para 2 enters the intrapartum unit. The most important nursing assessments are a. contraction pattern, amount of discomfort, and pregnancy history. b. fetal heart rate, maternal vital signs, and the woman's nearness to birth. c. identification of ruptured membranes, the woman's gravida and para, and her support person. d. last food intake, when labor began, and cultural practices the couple desires.

ANS: B All options describe relevant intrapartum nursing assessments; however, this focused assessment has priority. If the maternal and fetal conditions are normal and birth is not imminent, other assessments can be performed in an unhurried manner. This includes: gravida, para, support person, pregnancy history, pain assessment, last food intake, and cultural practices

With regard to a pregnant woman's anxiety and pain experience, nurses should be aware that a. even mild anxiety must be treated. b. severe anxiety increases tension, which increases pain, which in turn increases fear and anxiety, and so on. c. anxiety may increase the perception of pain, but it does not affect the mechanism of labor. d. women who have had a painful labor will have learned from the experience and have less anxiety the second time because of increased familiarity.

ANS: B Anxiety and pain reinforce each other in a negative cycle. Mild anxiety is normal for a woman in labor and likely needs no special treatment other than the standard reassurances. Anxiety increases muscle tension and ultimately can build sufficiently to slow the progress of labor. Unfortunately, an anxious, painful first labor is likely to carry over, through expectations and memories, into an anxious and painful experience in the second pregnancy

The role of the nurse with regard to informed consent is to a. inform the patient about the procedure and have her sign the consent form. b. act as a patient advocate and help clarify the procedure and the options. c. call the physician to see the patient. d. witness the signing of the consent form.

ANS: B Nurses play a part in the informed consent process by clarifying and describing procedures or by acting as the woman's advocate and asking the primary health care provider for further explanations. The physician is responsible for informing the woman of her options, explaining the procedure, and advising the patient about potential risk factors. The physician must be present to explain the procedure to the patient. However, the nurse's responsibilities go further than simply asking the physician to see the patient. The nurse may witness the signing of the consent form. However, depending on the state's guidelines, the woman's husband or another hospital health care employee may sign as witness.

After change-of-shift report the nurse assumes care of a multiparous patient in labor. The woman is complaining of pain that radiates to her abdominal wall, lower back, and buttocks and down her thighs. Before implementing a plan of care, the nurse should understand that this type of pain is a. visceral. b. referred. c. somatic. d. afterpain

ANS: B As labor progresses the woman often experiences referred pain. This occurs when pain that originates in the uterus radiates to the abdominal wall, the lumbosacral area of the back, the gluteal area, and thighs. The woman usually has pain only during a contraction and is free from pain between contractions. Visceral pain is that which predominates in the first stage of labor. This pain originates from cervical changes, distention of the lower uterine segment, and uterine ischemia. Visceral pain is located over the lower portion of the abdomen. Somatic pain is described as intense, sharp, burning, and well localized. This results from stretching of the perineal tissues and the pelvic floor. This occurs during the second stage of labor. Pain experienced during the third stage of labor or afterward during the early after birth period is uterine. This pain is very similar to that experienced in the first stage of labor.

While caring for the patient who requires an induction of labor, the nurse should be cognizant that a. ripening the cervix usually results in a decreased success rate for induction. b. labor sometimes can be induced with balloon catheters or laminaria tents. c. oxytocin is less expensive than prostaglandins and more effective but creates greater health risks. d. amniotomy can be used to make the cervix more favorable for labor.

ANS: B Balloon catheters or laminaria tents are mechanical means of ripening the cervix. Ripening the cervix, making it softer and thinner, increases the success rate of induced labor. Prostaglandin E1 is less expensive and more effective than oxytocin but carries a greater risk. Amniotomy is the artificial rupture of membranes, which is used to induce labor only when the cervix is already ripe

It is important for the nurse to develop a realistic birth plan with the pregnant woman in her care. The nurse can explain that a major advantage of nonpharmacologic pain management is a. greater and more complete pain relief is possible. b. no side effects or risks to the fetus are involved. c. the woman remains fully alert at all times. d. a more rapid labor is likely.

ANS: B Because nonpharmacologic pain management does not include analgesics, adjunct drugs, or anesthesia, it is harmless to the mother and the fetus. There is less pain relief with nonpharmacologic pain management during childbirth. The woman's alertness is not altered by medication; however, the increase in pain will decrease alertness. Pain management may or may not alter the length of labor. At times when pain is decreased, the mother relaxes and labor progresses at a quicker pace

Nurses should know some basic definitions concerning preterm birth, preterm labor, and low birth weight. For instance a. the terms preterm birth and low birth weight can be used interchangeably. b. preterm labor is defined as cervical changes and uterine contractions occurring between 20 and 37 weeks of pregnancy. c. low birth weight is anything below 3.7 lbs. d. in the United States early in this century, preterm birth accounted for 18% to 20% of all births.

ANS: B Before 20 weeks, it is not viable (miscarriage); after 37 weeks, it can be considered term. Although these terms are used interchangeably, they have different meanings: preterm birth describes the length of gestation (37 weeks) regardless of weight; low birth weight describes weight only (2500 g or less) at the time of birth, whenever it occurs. Low birth weight is anything less than 2500 g, or about 5.5 lbs. In 2003 the preterm birth rate in the United States was 12.3%, but it is increasing in frequency.

Which statement is the best rationale for assessing maternal vital signs between contractions? a. During a contraction, assessing fetal heart rates is the priority. b. Maternal circulating blood volume increases temporarily during contractions. c. Maternal blood flow to the heart is reduced during contractions. d. Vital signs taken during contractions are not accurate

ANS: B During uterine contractions, blood flow to the placenta temporarily stops, causing a relative increase in the mother's blood volume, which in turn temporarily increases blood pressure and slows pulse. It is important to monitor fetal response to contractions; however, this question is concerned with the maternal vital signs. Maternal blood flow is increased during a contraction. Vital signs are altered by contractions but are considered accurate for that period of time

With regard to systemic analgesics administered during labor, nurses should be aware that a. systemic analgesics cross the maternal blood-brain barrier as easily as they do the fetal blood-brain barrier. b. effects on the fetus and newborn can include decreased alertness and delayed sucking. c. intramuscular (IM) administration is preferred over intravenous (IV) administration. d. IV patient-controlled analgesia (PCA) results in increased use of an analgesic

ANS: B Effects depend on the specific drug given, the dosage, and the timing. Systemic analgesics cross the fetal blood-brain barrier more readily than the maternal blood-brain barrier. IV administration is preferred over IM administration because the drug acts faster and more predictably. PCA results in decreased use of an analgesic

The slight overlapping of cranial bones or shaping of the fetal head during labor is called a. lightening. b. molding. c. Ferguson reflex. d. Valsalva maneuver.

ANS: B Fetal head formation is called molding. Molding also permits adaptation to various diameters of the maternal pelvis. Lightening is the mother's sensation of decreased abdominal distention, which usually occurs the week before labor. The Ferguson reflex is the contraction urge of the uterus after stimulation of the cervix. The Valsalva maneuver describes conscious pushing during the second stage of labor

With regard to spinal and epidural (block) anesthesia, nurses should know that a. this type of anesthesia is commonly used for cesarean births but is not suitable for vaginal births. b. a high incidence of after-birth headache is seen with spinal blocks. c. epidural blocks allow the woman to move freely. d. spinal and epidural blocks are never used together.

ANS: B Headaches may be prevented or mitigated to some degree by a number of methods. Spinal blocks may be used for vaginal births, but the woman must be assisted through labor. Epidural blocks limit the woman's ability to move freely. Combined use of spinal and epidural blocks is becoming increasingly popular.

What correctly matches the type of deceleration with its likely cause? a. Early deceleration—umbilical cord compression b. Late deceleration—uteroplacental inefficiency c. Variable deceleration—head compression d. Prolonged deceleration—cause unknown

ANS: B Late deceleration is caused by uteroplacental inefficiency. Early deceleration is caused by head compression. Variable deceleration is caused by umbilical cord compression. Prolonged deceleration has a variety of either benign or critical causes.

What three measures should the nurse implement to provide intrauterine resuscitation? Select the response that best indicates the priority of actions that should be taken. a. Call the provider, reposition the mother, and perform a vaginal examination. b. Reposition the mother, increase intravenous (IV) fluid, and provide oxygen via face mask. c. Administer oxygen to the mother, increase IV fluid, and notify the care provider. d. Perform a vaginal examination, reposition the mother, and provide oxygen via face mask.

ANS: B Repositioning the mother, increasing intravenous (IV) fluid, and providing oxygen via face mask are correct nursing actions for intrauterine resuscitation. The nurse should initiate intrauterine resuscitation in an ABC manner, similar to basic life support. The first priority is to open the maternal and fetal vascular systems by repositioning the mother for improved perfusion. The second priority is to increase blood volume by increasing the IV fluid. The third priority is to optimize oxygenation of the circulatory volume by providing oxygen via face mask. If these interventions do not resolve the fetal heart rate issue quickly, the primary provider should be notified immediately

A first-time mother is concerned about the type of medications she will receive during labor. She is in a fair amount of pain and is nauseous. In addition, she appears to be very anxious. You explain that opioid analgesics are often used with sedatives because a. ―The two together work the best for you and your baby.‖ b. ―Sedatives help the opioid work better, and they also will assist you to relax and relieve your nausea.‖ c. ―They work better together so you can sleep until you have the baby.‖ d. ―This is what the doctor has ordered for you.‖

ANS: B Sedatives can be used to reduce the nausea and vomiting that often accompany opioid use. In addition, some ataractics reduce anxiety and apprehension and potentiate the opioid analgesic affects. A potentiator may cause the two drugs to work together more effectively, but it does not ensure maternal or fetal complications will not occur. Sedation may be a related effect of some ataractics, but it is not the goal. Furthermore, a woman is unlikely to be able to sleep through transitional labor and birth. ―This is what the doctor has ordered for you‖ may be true, but it is not an acceptable comment for the nurse to make

After an emergency birth, the nurse encourages the woman to breastfeed her newborn. The primary purpose of this activity is to a. facilitate maternal-newborn interaction. b. stimulate the uterus to contract. c. prevent neonatal hypoglycemia. d. initiate the lactation cycle

ANS: B Stimulation of the nipples through breastfeeding or manual stimulation causes the release of oxytocin and prevents maternal hemorrhage. Breastfeeding facilitates maternal-newborn interaction, but it is not the primary reason a woman is encouraged to breastfeed after an emergency birth. The primary intervention for preventing neonatal hypoglycemia is thermoregulation. Cold stress can result in hypoglycemia. The woman is encouraged to breastfeed after an emergency birth to stimulate the release of oxytocin, which prevents hemorrhage. Breastfeeding is encouraged to initiate the lactation cycle, but it is not the primary reason for this activity after an emergency birth

The nurse providing care for a woman with preterm labor who is receiving terbutaline would include which intervention to identify side effects of the drug? a. Assessing deep tendon reflexes (DTRs) b. Assessing for chest discomfort and palpitations c. Assessing for bradycardia d. Assessing for hypoglycemia

ANS: B Terbutaline is a 2-adrenergic agonist that affects the cardiopulmonary and metabolic systems of the mother. Signs of cardiopulmonary decompensation would include chest pain and palpitations. Assessing DTRs would not address these concerns. 2-Adrenergic agonist drugs cause tachycardia, not bradycardia. The metabolic effect leads to hyperglycemia, not hypoglycemia

Nurses can help their patients by keeping them informed about the distinctive stages of labor. Which description of the phases of the first stage of labor is accurate? a. Latent: Mild, regular contractions; no dilation; bloody show; duration of 2 to 4 hours b. Active: Moderate, regular contractions; 4- to 7-cm dilation; duration of 3 to 6 hours c. Lull: No contractions; dilation stable; duration of 20 to 60 minutes d. Transition: Very strong but irregular contractions; 8- to 10-cm dilation; duration of 1 to 2 hours

ANS: B The active phase is characterized by moderate, regular contractions; 4- to 7-cm dilation; and a duration of 3 to 6 hours. The latent phase is characterized by mild-to-moderate, irregular contractions; dilation up to 3 cm; brownish-to-pale pink mucus, and a duration of 6 to 8 hours. No official ―lull‖ phase exists in the first stage. The transition phase is characterized by strong to very strong, regular contractions; 8- to 10-cm dilation; and a duration of 20 to 40 minutes

The nurse should be aware that an effective plan to achieve adequate pain relief without maternal risk is most effective if a. the mother gives birth without any analgesic or anesthetic. b. the mother and family's priorities and preferences are incorporated into the plan. c. the primary health care provider decides the best pain relief for the mother and family. d. the nurse informs the family of all alternative methods of pain relief available in the hospital setting.

ANS: B The assessment of the woman, her fetus, and her labor is a joint effort of the nurse and the primary health care providers, who consult with the woman about their findings and recommendations. The needs of each woman are different and many factors must be considered before a decision is made whether pharmacologic methods, nonpharmacologic methods, or a combination of the two will be used to manage labor pain.

During labor a fetus with an average heart rate of 135 beats/min over a 10-minute period would be considered to have a. bradycardia. b. a normal baseline heart rate. c. tachycardia. d. hypoxia

ANS: B The baseline heart rate is measured over 10 minutes; a normal range is 110 to 160 beats/min. Bradycardia is a fetal heart rate (FHR) below 110 beats/min for 10 minutes or longer. Tachycardia is an FHR over 160 beats/min for 10 minutes or longer. Hypoxia is an inadequate supply of oxygen; no indication of this condition exists with a baseline heart rate in the normal range

To teach patients about the process of labor adequately, the nurse knows that which event is the best indicator of true labor? a. Bloody show b. Cervical dilation and effacement c. Fetal descent into the pelvic inlet d. Uterine contractions every 7 minutes

ANS: B The conclusive distinction between true and false labor is that contractions of true labor cause progressive change in the cervix. Bloody show can occur before true labor. Fetal descent can occur before true labor. False labor may have contractions that occur this frequently; however, this is usually inconsistent

The nurse has received report regarding her patient in labor. The woman's last vaginal examination was recorded as 3 cm, 30%, and -2. The nurse's interpretation of this assessment is that a. the cervix is effaced 3 cm, it is dilated 30%, and the presenting part is 2 cm above the ischial spines. b. the cervix is 3 cm dilated, it is effaced 30%, and the presenting part is 2 cm above the ischial spines. c. the cervix is effaced 3 cm, it is dilated 30%, and the presenting part is 2 cm below the ischial spines. d. the cervix is dilated 3 cm, it is effaced 30%, and the presenting part is 2 cm below the ischial spines.

ANS: B The correct description of the vaginal examination for this woman in labor is the cervix is 3 cm dilated, it is effaced 30%, and the presenting part is 2 cm above the ischial spines. The sterile vaginal examination is recorded as centimeters of cervical dilation, percentage of cervical dilation, and the relationship of the presenting part to the ischial spines (either above or below).

Through vaginal examination the nurse determines that a woman is 4 cm dilated, and the external fetal monitor shows uterine contractions every 3.5 to 4 minutes. The nurse would report this as a. first stage, latent phase. b. first stage, active phase. c. first stage, transition phase. d. second stage, latent phase

ANS: B The first stage, active phase of maternal progress indicates that the woman is in the active phase of the first stage of labor. During the latent phase of the first stage of labor, the expected maternal progress would be 0 to 3 cm dilation with contractions every 5 to 30 minutes. During the transition phase of the first stage of labor, the expected maternal progress is 8 to 10 cm dilation with contractions every 2 to 3 minutes. During the latent phase of the second stage of labor, the woman is completely dilated and experiences a restful period of ―laboring down.‖

A patient whose cervix is dilated to 5 cm is considered to be in which phase of labor? a. Latent phase b. Active phase c. Second stage d. Third stage

ANS: B The latent phase is from the beginning of true labor until 3 cm of cervical dilation. The active phase of labor is characterized by cervical dilation of 4 to 7 cm. The second stage of labor begins when the cervix is completely dilated until the birth of the baby. The third stage of labor is from the birth of the baby until the expulsion of the placenta. This patient is in the active phase of labor.

For women who have a history of sexual abuse, a number of traumatic memories may be triggered during labor. The woman may fight the labor process and react with pain or anger. Alternately, she may become a passive player and emotionally absent herself from the process. The nurse is in a unique position of being able to assist the patient to associate the sensations of labor with the process of childbirth and not the past abuse. The nurse can implement a number of care measures to help the patient view the childbirth experience in a positive manner. Which intervention would be key for the nurse to use while providing care? a. Telling the patient to relax and that it won't hurt much. b. Limiting the number of procedures that invade her body. c. Reassuring the patient that as the nurse you know what is best. d. Allowing unlimited care providers to be with the patient

ANS: B The number of invasive procedures such as vaginal examinations, internal monitoring, and intravenous therapy should be limited as much as possible. The nurse should always avoid words and phrases that may result in the patient's recalling the phrases of her abuser (e.g., ―Relax, this won't hurt‖ or ―Just open your legs.‖) The woman's sense of control should be maintained at all times. The nurse should explain procedures at the patient's pace and wait for permission to proceed. Protecting the patient's environment by providing privacy and limiting the number of staff who observe the patient will help to make her feel safe.

Nurses alert to signs of the onset of the second stage of labor can be certain that this stage has begun when a. the woman has a sudden episode of vomiting. b. the nurse is unable to feel the cervix during a vaginal examination. c. bloody show increases. d. the woman involuntarily bears down.

ANS: B The only certain objective sign that the second stage has begun is the inability to feel the cervix because it is fully dilated and effaced. Vomiting, an increase in bloody show, and involuntary bearing down are only suggestions of second-stage labor

As relates to fetal positioning during labor, nurses should be aware that a. position is a measure of the degree of descent of the presenting part of the fetus through the birth canal. b. birth is imminent when the presenting part is at +4 to +5 cm below the spine. c. the largest transverse diameter of the presenting part is the suboccipitobregmatic diameter. d. engagement is the term used to describe the beginning of labor.

ANS: B The station of the presenting part should be noted at the beginning of labor so that the rate of descent can be determined. Position is the relation of the presenting part of the fetus to the four quadrants of the mother's pelvis; station is the measure of degree of descent. The largest diameter usually is the biparietal diameter. The suboccipitobregmatic diameter is the smallest, although one of the most critical. Engagement often occurs in the weeks just before labor in nulliparas and before or during labor in multiparas.

To assist the woman after delivery of the infant, the nurse knows that the blood patch is used after spinal anesthesia to relieve a. hypotension. b. headache. c. neonatal respiratory depression. d. loss of movement.

ANS: B The subarachnoid block may cause a postspinal headache resulting from loss of cerebrospinal fluid from the puncture in the dura. When blood is injected into the epidural space in the area of the dural puncture, it forms a seal over the hole to stop leaking of cerebrospinal fluid. Hypotension is prevented by increasing fluid volume before the procedure. Neonatal respiratory depression is not an expected outcome with spinal anesthesia. Loss of movement is an expected outcome of spinal anesthesia.

If an opioid antagonist is administered to a laboring woman, she should be told that a. her pain will decrease. b. her pain will return. c. she will feel less anxious. d. she will no longer feel the urge to push.

ANS: B The woman should be told that the pain that was relieved by the opioid analgesic will return with administration of the opioid antagonist. Opioid antagonists, such as Narcan, promptly reverse the central nervous system (CNS) depressant effects of opioids. In addition, the antagonist counters the effect of the stress-induced levels of endorphins. An opioid antagonist is especially valuable if labor is more rapid than expected and birth is anticipated when the opioid is at its peak effect

The nurse, caring for a patient whose labor is being augmented with oxytocin, recognizes that the oxytocin should be discontinued immediately if there is evidence of a. uterine contractions occurring every 8 to 10 minutes. b. a fetal heart rate (FHR) of 180 with absence of variability. c. the patient's needing to void. d. rupture of the patient's amniotic membranes.

ANS: B This FHR is nonreassuring. The oxytocin should be discontinued immediately, and the physician should be notified. The oxytocin should be discontinued if uterine hyperstimulation occurs. Uterine contractions that are occurring every 8 to 10 minutes do not qualify as hyperstimulation. The patient's needing to void is not an indication to discontinue the oxytocin induction immediately or to call the physician. Unless a change occurs in the FHR pattern that is nonreassuring or the patient experiences uterine hyperstimulation, the oxytocin does not need to be discontinued. The physician should be notified that the patient's membranes have ruptured.

The nurse providing care for the laboring woman realizes that variable fetal heart rate (FHR) decelerations are caused by a. altered fetal cerebral blood flow. b. umbilical cord compression. c. uteroplacental insufficiency. d. fetal hypoxemia

ANS: B Variable decelerations can occur any time during the uterine contracting phase and are caused by compression of the umbilical cord. Altered fetal cerebral blood flow would result in early decelerations in the FHR. Uteroplacental insufficiency would result in late decelerations in the FHR. Fetal hypoxemia would result in tachycardia initially and then bradycardia if hypoxia continues.

A woman who is 39 weeks pregnant expresses fear about her impending labor and how she will manage. The nurse's best response is a. ―Don't worry about it. You'll do fine.‖ b. ―It's normal to be anxious about labor. Let's discuss what makes you afraid.‖ c. ―Labor is scary to think about, but the actual experience isn't.‖ d. ―You can have an epidural. You won't feel anything.‖

ANS: B ―It's normal to be anxious about labor. Let's discuss what makes you afraid‖ allows the woman to share her concerns with the nurse and is a therapeutic communication tool. ―Don't worry about it. You'll do fine‖ negates the woman's fears and is not therapeutic. ―Labor is scary to think about, but the actual experience isn't‖ negates the woman's fears and offers a false sense of security. It is not true that every woman may have an epidural. A number of criteria must be met for use of an epidural. Furthermore, many women still experience the feeling of pressure with an epidural.

A new patient and her partner arrive in the labor, delivery, recovery, and after birth unit for the birth of their first child. You apply the electronic fetal monitor (EFM) to the woman. Her partner asks you to explain what is printing on the graph, referring to the EFM strip. He wants to know what the baby's heart rate should be. Your best response is a. ―Don't worry about that machine; that's my job.‖ b. ―The top line graphs the baby's heart rate. Generally, the heart rate is between 110 and 160. The heart rate will fluctuate in response to what is happening during labor.‖ c. ―The top line graphs the baby's heart rate, and the bottom line lets me know how strong the contractions are.‖ d. ―Your doctor will explain all of that later.

ANS: B ―The top line graphs the baby's heart rate. Generally, the heart rate is between 110 and 160. The heart rate will fluctuate in response to what is happening during labor‖ educates the partner about fetal monitoring and provides support and information to alleviate his fears. ―Don't worry about that machine; that's my job‖ discredits the partner's feelings and does not provide the teaching he is requesting. ―The top line graphs the baby's heart rate, and the bottom line lets me know how strong the contractions are‖ provides inaccurate information and does not address the partner's concerns about the fetal heart rate. The EFM graphs the frequency and duration of the contractions, not the intensity. Nurses should take every opportunity to provide patient and family teaching, especially when information is requested.

Maternal hypotension is a potential side effect of regional anesthesia and analgesia. What nursing interventions could you use to raise the patient's blood pressure? (Select all that apply.) a. Place the woman in a supine position. b. Place the woman in a lateral position. c. Increase intravenous (IV) fluids. d. Administer oxygen. e. Perform a vaginal examination

ANS: B, C, D Nursing interventions for maternal hypotension arising from analgesia or anesthesia include turning the woman to a lateral position, increasing IV fluids, administering oxygen via face mask, elevating the woman's legs, notifying the physician, administering an IV vasopressor, and monitoring the maternal and fetal status at least every 5 minutes until these are stable. Placing the patient in a supine position would cause venous compression, thereby limiting blood flow to and oxygenation of the placenta and fetus. A sterile vaginal examination has no bearing on maternal blood pressure.

A tiered system of categorizing FHR has been recommended by regulatory agencies. Nurses, midwives, and physicians who care for women in labor must have a working knowledge of fetal monitoring standards and understand the significance of each category. These categories include (Select all that apply.) a. reassuring. b. Category I. c. Category II. d. nonreassuring. e. Category III.

ANS: B, C, E The three-tiered system of FHR tracings include Category I, II, and III. Category I is a normal tracing requiring no action. Category II FHR tracings are indeterminate. This category includes tracings that do not meet Category I or III criteria. Category III tracings are abnormal and require immediate intervention

Which nursing assessment indicates that a woman who is in second-stage labor is almost ready to give birth? a. The fetal head is felt at 0 station during vaginal examination. b. Bloody mucus discharge increases. c. The vulva bulges and encircles the fetal head. d. The membranes rupture during a contraction

ANS: C A bulging vulva that encircles the fetal head describes crowning, which occurs shortly before birth. Birth of the head occurs when the station is +4. A 0 station indicates engagement. Bloody show occurs throughout the labor process and is not an indication of an imminent birth. Rupture of membranes can occur at any time during the labor process and does not indicate an imminent birth

The nurse thoroughly dries the infant immediately after birth primarily to a. stimulate crying and lung expansion. b. remove maternal blood from the skin surface. c. reduce heat loss from evaporation. d. increase blood supply to the hands and feet.

ANS: C Infants are wet with amniotic fluid and blood at birth, and this accelerates evaporative heat loss. The primary purpose of drying the infant is to prevent heat loss. Rubbing the infant does stimulate crying; however, it is not the main reason for drying the infant. This process does not remove all the maternal blood.

It is paramount for the obstetric nurse to understand the regulatory procedures and criteria for admitting a woman to the hospital labor unit. Which guideline is an important legal requirement of maternity care? a. The patient is not considered to be in true labor (according to the Emergency Medical Treatment and Active Labor Act [EMTALA]) until a qualified health care provider says she is. b. The woman can have only her male partner or predesignated ―doula‖ with her at assessment. c. The patient's weight gain is calculated to determine whether she is at greater risk for cephalopelvic disproportion (CPD) and cesarean birth. d. The nurse may exchange information about the patient with family members.

ANS: C According to EMTALA, a woman is entitled to active labor care and is presumed to be in ―true‖ labor until a qualified health care provider certifies otherwise. A woman can have anyone she wishes present for her support. The risk for CPD is especially great for petite women or those who have gained 16 kg or more. All patients should have their weight and BMI calculated on admission. This is part of standard nursing care on a maternity unit and not a regulatory concern. According to the Health Insurance Portability and Accountability Act (HIPAA), the patient must give consent for others to receive any information related to her condition.

When assessing a woman in labor, the nurse is aware that the relationship of the fetal body parts to one another is called fetal a. lie. b. presentation. c. attitude. d. position

ANS: C Attitude is the relation of the fetal body parts to one another. Lie is the relation of the long axis (spine) of the fetus to the long axis (spine) of the mother. Presentation refers to the part of the fetus that enters the pelvic inlet first and leads through the birth canal during labor at term. Position is the relation of the presenting part to the four quadrants of the mother's pelvis

The priority nursing care associated with an oxytocin (Pitocin) infusion is a. measuring urinary output. b. increasing infusion rate every 30 minutes. c. monitoring uterine response. d. evaluating cervical dilation.

ANS: C Because of the risk of hyperstimulation, which could result in decreased placental perfusion and uterine rupture, the nurse's priority intervention is monitoring uterine response. Monitoring urinary output is also important; however, it is not the top priority during the administration of Pitocin. The infusion rate may be increased after proper assessment that it is an appropriate interval to do so. Monitoring labor progression is the standard of care for all labor patients.

As relates to the use of tocolytic therapy to suppress uterine activity, nurses should be aware that a. the drugs can be given efficaciously up to the designated beginning of term at 37 weeks. b. there are no important maternal (as opposed to fetal) contraindications. c. its most important function is to afford the opportunity to administer antenatal glucocorticoids. d. if the patient develops pulmonary edema while receiving tocolytics, intravenous (IV) fluids should be given.

ANS: C Buying time for antenatal glucocorticoids to accelerate fetal lung development may be the best reason to use tocolytics. Once the pregnancy has reached 34 weeks, the risks of tocolytic therapy outweigh the benefits. There are important maternal contraindications to tocolytic therapy. Tocolytic-induced edema can be caused by IV fluids

A nulliparous woman who has just begun the second stage of her labor would most likely a. experience a strong urge to bear down. b. show perineal bulging. c. feel tired yet relieved that the worst is over. d. show an increase in bright red bloody show.

ANS: C Common maternal behaviors during the latent phase of the second stage of labor include feeling a sense of accomplishment and optimism because ―the worst is over.‖ During the latent phase of the second stage of labor, the urge to bear down often is absent or only slight during the acme of contractions. Perineal bulging occurs during the transition phase of the second stage of labor, not at the beginning of the second stage. An increase in bright red bloody show occurs during the descent phase of the second stage of labor.

Which nursing assessment indicates that a woman who is in second-stage labor is almost ready to give birth? a. The fetal head is felt at 0 station during vaginal examination. b. Bloody mucus discharge increases. c. The vulva bulges and encircles the fetal head. d. The membranes rupture during a contraction.

ANS: C During the active pushing (descent) phase, the woman has strong urges to bear down as the presenting part of the fetus descends and presses on the stretch receptors of the pelvic floor. The vulva stretches and begins to bulge encircling the fetal head. Birth of the head occurs when the station is +4. A 0 station indicates engagement. Bloody show occurs throughout the labor process and is not an indication of an imminent birth. Rupture of membranes can occur at any time during the labor process and does not indicate an imminent birth

The nurse knows that the second stage of labor, the descent phase, has begun when a. the amniotic membranes rupture. b. the cervix cannot be felt during a vaginal examination. c. the woman experiences a strong urge to bear down. d. the presenting part is below the ischial spines

ANS: C During the descent phase of the second stage of labor, the woman may experience an increase in the urge to bear down. Rupture of membranes has no significance in determining the stage of labor. The second stage of labor begins with full cervical dilation. Many women may have an urge to bear down when the presenting part is below the level of the ischial spines. This can occur during the first stage of labor, as early as 5-cm dilation

While providing care to a patient in active labor, the nurse should instruct the woman that a. the supine position commonly used in the United States increases blood flow. b. the ―all fours‖ position, on her hands and knees, is hard on her back. c. frequent changes in position will help relieve her fatigue and increase her comfort. d. in a sitting or squatting position, her abdominal muscles will have to work harder

ANS: C Frequent position changes relieve fatigue, increase comfort, and improve circulation. Blood flow can be compromised in the supine position; any upright position benefits cardiac output. The ―all fours‖ position is used to relieve backache in certain situations. In a sitting or squatting position, the abdominal muscles work in greater harmony with uterine contractions.

The laboring woman who imagines her body opening to let the baby out is using a mental technique called a. dissociation. b. effleurage. c. imagery. d. distraction

ANS: C Imagery is a technique of visualizing images that will assist the woman in coping with labor. Dissociation helps the woman learn to relax all muscles except those that are working. Effleurage is self-massage. Distraction can be used in the early latent phase by having the woman engage in another activity.

Prepidil (prostaglandin gel) has been ordered for a pregnant woman at 43 weeks of gestation. The nurse recognizes that this medication will be administered to a. enhance uteroplacental perfusion in an aging placenta. b. increase amniotic fluid volume. c. ripen the cervix in preparation for labor induction. d. stimulate the amniotic membranes to rupture.

ANS: C It is accurate to state that Prepidil will be administered to ripen the cervix in preparation for labor induction. It is not administered to enhance uteroplacental perfusion in an aging placenta, increase amniotic fluid volume, or stimulate the amniotic membranes to rupture.

The nurse caring for the woman in labor should understand that maternal hypotension can result in a. early decelerations. b. fetal dysrhythmias. c. uteroplacental insufficiency. d. spontaneous rupture of membranes.

ANS: C Low maternal blood pressure reduces placental blood flow during uterine contractions and results in fetal hypoxemia. Maternal hypotension is not associated with early decelerations, fetal dysrhythmias, or spontaneous rupture of membranes

The nurse caring for the woman in labor should understand that increased variability of the fetal heart rate may be caused by a. narcotics. b. barbiturates. c. methamphetamines. d. tranquilizers.

ANS: C Narcotics, barbiturates, and tranquilizers may be causes of decreased variability; methamphetamines may cause increased variability.

Surgical, medical, or mechanical methods may be used for labor induction. Which technique is considered a mechanical method of induction? a. Amniotomy b. Intravenous Pitocin c. Transcervical catheter d. Vaginal insertion of prostaglandins

ANS: C Placement of a balloon-tipped Foley catheter into the cervix is a mechanical method of induction. Other methods to expand and gradually dilate the cervix include hydroscopic dilators such as laminaria tents (made from desiccated seaweed), or Lamicel (contains magnesium sulfate). Amniotomy is a surgical method of augmentation and induction. Intravenous Pitocin and insertion of prostaglandins are medical methods of induction

In planning for home care of a woman with preterm labor, which concern must the nurse address? a. Nursing assessments will be different from those done in the hospital setting. b. Restricted activity and medications will be necessary to prevent recurrence of preterm labor. c. Prolonged bed rest may cause negative physiologic effects. d. Home health care providers will be necessary

ANS: C Prolonged bed rest may cause adverse effects such as weight loss, loss of appetite, muscle wasting, weakness, bone demineralization, decreased cardiac output, risk for thrombophlebitis, alteration in bowel functions, sleep disturbance, and prolonged after birth recovery. Nursing assessments will differ somewhat from those performed in the acute care setting, but this is not the concern that needs to be addressed. Restricted activity and medication may prevent preterm labor, but not in all women. In addition, the plan of care is individualized to meet the needs of each woman. Many women will receive home health nurse visits, but care is individualized for each woman.

The nurse recognizes that a woman is in true labor when she states a. ―I passed some thick, pink mucus when I urinated this morning.‖ b. ―My bag of waters just broke.‖ c. ―The contractions in my uterus are getting stronger and closer together.‖ d. ―My baby dropped, and I have to urinate more frequently now

ANS: C Regular, strong contractions with the presence of cervical change indicate that the woman is experiencing true labor. Loss of the mucous plug (operculum) often occurs during the first stage of labor or before the onset of labor, but it is not the indicator of true labor. Spontaneous rupture of membranes often occurs during the first stage of labor, but it is not the indicator of true labor. The presenting part of the fetus typically becomes engaged in the pelvis at the onset of labor, but this is not the indicator of true labor.

At 1 minute after birth, the nurse assesses the newborn to assign an Apgar score. The apical heart rate is 110 bpm, and the infant is crying vigorously with the limbs flexed. The infant's trunk is pink, but the hands and feet are blue. What is the correct Apgar score for this infant? a. 7 b. 8 c. 9 d. 10

ANS: C The Apgar score is 9 because 1 point is deducted from the total score of 10 for the infant's blue hands and feet. The baby received 2 points for each of the categories except color. Because the infant's hands and feet were blue, this category is given a grade of 1.

According to standard professional thinking, nurses should auscultate the fetal heart rate (FHR) a. every 15 minutes in the active phase of the first stage of labor in the absence of risk factors. b. every 20 minutes in the second stage, regardless of whether risk factors are present. c. before and after ambulation and rupture of membranes. d. more often in a woman's first pregnancy.

ANS: C The FHR should be auscultated before and after administration of medications and induction of anesthesia. In the active phase of the first stage of labor, the FHR should be auscultated every 30 minutes if no risk factors are involved; with risk factors it should be auscultated every 15 minutes. In the second stage of labor the FHR should be auscultated every 15 minutes if no risk factors are involved; with risk factors it should be auscultated every 5 minutes. The fetus of a first-time mother is automatically at greater risk

Which description of the phases of the second stage of labor is accurate? a. Latent phase: Feeling sleepy, fetal station 2+ to 4+, duration 30 to 45 minutes b. Active phase: Overwhelmingly strong contractions, Ferguson reflux activated, duration 5 to 15 minutes c. Descent phase: Significant increase in contractions, Ferguson reflux activated, average duration varied d. Transitional phase: Woman ―laboring down,‖ fetal station 0, duration 15 minutes

ANS: C The descent phase begins with a significant increase in contractions; the Ferguson reflex is activated, and the duration varies, depending on a number of factors. The latent phase is the lull, or ―laboring down,‖ period at the beginning of the second stage. It lasts 10 to 30 minutes on average. The second stage of labor has no active phase. The transition phase is the final phase in the second stage of labor; contractions are strong and painful.

The maternity nurse understands that as the uterus contracts during labor, maternal-fetal exchange of oxygen and waste products a. continues except when placental functions are reduced. b. increases as blood pressure decreases. c. diminishes as the spiral arteries are compressed. d. is not significantly affected.

ANS: C Uterine contractions during labor tend to decrease circulation through the spiral electrodes and subsequent perfusion through the intervillous space. The maternal blood supply to the placenta gradually stops with contractions. The exchange of oxygen and waste products decreases. The exchange of oxygen and waste products is affected by contractions

When assessing the fetus using Leopold maneuvers, the nurse feels a round, firm, movable fetal part in the fundal portion of the uterus and a long, smooth surface in the mother's right side close to midline. What is the likely position of the fetus? a. ROA b. LSP c. RSA d. LOA

ANS: C The fetus is positioned anteriorly in the right side of the maternal pelvis with the sacrum as the presenting part. RSA is the correct three-letter abbreviation to indicate this fetal position. The first letter indicates the presenting part in either the right or left side of the maternal pelvis. The second letter indicates the anatomic presenting part of the fetus. The third letter stands for the location of the presenting part in relation to the anterior, posterior, or transverse portion of the maternal pelvis. Palpation of a round, firm fetal part in the fundal portion of the uterus would be the fetal head, indicating that the fetus is in a breech position with the sacrum as the presenting part in the maternal pelvis. Palpation of the fetal spine along the mother's right side denotes the location of the presenting part in the mother's pelvis. The ability to palpate the fetal spine indicates that the fetus is anteriorly positioned in the maternal pelvis

A woman in labor has just received an epidural block. The most important nursing intervention is to a. limit parenteral fluids. b. monitor the fetus for possible tachycardia. c. monitor the maternal blood pressure for possible hypotension. d. monitor the maternal pulse for possible bradycardia.

ANS: C The most important nursing intervention for a woman who has received an epidural block is to monitor the maternal blood pressure frequently for signs of hypotension. Intravenous fluids are increased for a woman receiving an epidural, to prevent hypotension. The nurse observes for signs of fetal bradycardia. The nurse monitors for signs of maternal tachycardia secondary to hypotension.

With regard to factors that affect how the fetus moves through the birth canal, nurses should be aware that a. the fetal attitude describes the angle at which the fetus exits the uterus. b. of the two primary fetal lies, the horizontal lie is that in which the long axis of the fetus is parallel to the long axis of the mother. c. the normal attitude of the fetus is called general flexion. d. the transverse lie is preferred for vaginal birth.

ANS: C The normal attitude of the fetus is general flexion. The fetal attitude is the relation of fetal body parts to one another. The horizontal lie is perpendicular to the mother; in the longitudinal (or vertical) lie the long axes of the fetus and the mother are parallel. Vaginal birth cannot occur if the fetus stays in a transverse lie

Which action is correct when palpation is used to assess the characteristics and pattern of uterine contractions? a. Place the hand on the abdomen below the umbilicus and palpate uterine tone with the fingertips. b. Determine the frequency by timing from the end of one contraction to the end of the next contraction. c. Evaluate the intensity by pressing the fingertips into the uterine fundus. d. Assess uterine contractions every 30 minutes throughout the first stage of labor.

ANS: C The nurse or primary care provider may assess uterine activity by palpating the fundal section of the uterus using the fingertips. Many women may experience labor pain in the lower segment of the uterus that may be unrelated to the firmness of the contraction detectable in the uterine fundus. The frequency of uterine contractions is determined by palpating from the beginning of one contraction to the beginning of the next contraction. Assessment of uterine activity is performed in intervals based on the stage of labor. As labor progresses this assessment is performed more frequently

With regard to the turns and other adjustments of the fetus during the birth process, known as the mechanism of labor, nurses should be aware that a. the seven critical movements must progress in a more or less orderly sequence. b. asynclitism sometimes is achieved by means of the Leopold maneuver. c. the effects of the forces determining descent are modified by the shape of the woman's pelvis and the size of the fetal head. d. at birth the baby is said to achieve ―restitution‖ (i.e., a return to the C-shape of the womb)

ANS: C The size of the maternal pelvis and the ability of the fetal head to mold also affect the process. The seven identifiable movements of the mechanism of labor occur in combinations simultaneously, not in precise sequences. Asynclitism is the deflection of the baby's head; the Leopold maneuver is a means of judging descent by palpating the mother's abdomen. Restitution is the rotation of the baby's head after the infant is born

What is an advantage of external electronic fetal monitoring? a. The ultrasound transducer can accurately measure short-term variability and beat-to-beat changes in the fetal heart rate. b. The tocotransducer can measure and record the frequency, regularity, intensity, and approximate duration of uterine contractions (UCs). c. The tocotransducer is especially valuable for measuring uterine activity during the first stage of labor. d. Once correctly applied by the nurse, the transducer need not be repositioned even when the woman changes positions

ANS: C The tocotransducer is especially valuable for measuring uterine activity during the first stage of labor, particularly when the membranes are intact. Short-term changes cannot be measured with this technology. The tocotransducer cannot measure and record the intensity of UCs. The transducer must be repositioned when the woman or fetus changes position.

A woman in labor is breathing into a mouthpiece just before the start of her regular contractions. As she inhales, a valve opens, and gas is released. She continues to inhale the gas slowly and deeply until the contraction starts to subside. When the inhalation stops, the valve closes. This procedure is a. not used much anymore. b. likely to be used in the second stage of labor but not in the first stage. c. an application of nitrous oxide. d. a prelude to cesarean birth.

ANS: C This is an application of nitrous oxide, which could be used in either the first or second stage of labor (or both) as part of the preparation for a vaginal birth. Nitrous oxide is self-administered and found to be very helpful.

The nurse teaches a pregnant woman about the characteristics of true labor contractions. The nurse evaluates the woman's understanding of the instructions when she states, ―True labor contractions will a. subside when I walk around.‖ b. cause discomfort over the top of my uterus.‖ c. continue and get stronger even if I relax and take a shower.‖ d. remain irregular but become stronger

ANS: C True labor contractions occur regularly, becoming stronger, lasting longer, and occurring closer together. They may become intense during walking and continue despite comfort measures. Typically, true labor contractions are felt in the lower back, radiating to the lower portion of the abdomen. During false labor, contractions tend to be irregular and felt in the abdomen above the navel. Typically, the contractions often stop with walking or a change of position.

The nurse providing care for the laboring woman should understand that late fetal heart rate (FHR) decelerations are the result of a. altered cerebral blood flow. b. umbilical cord compression. c. uteroplacental insufficiency. d. meconium fluid.

ANS: C Uteroplacental insufficiency would result in late decelerations in the FHR. Altered fetal cerebral blood flow would result in early decelerations in the FHR. Umbilical cord compression would result in variable decelerations in the FHR. Meconium-stained fluid may or may not produce changes in the fetal heart rate, depending on the gestational age of the fetus and whether other causative factors associated with fetal distress are present.

A woman is having her first child. She has been in labor for 15 hours. Two hours ago her vaginal examination revealed the cervix to be dilated to 5 cm and 100% effaced, and the presenting part was at station 0. Five minutes ago her vaginal examination indicated that there had been no change. What abnormal labor pattern is associated with this description? a. Prolonged latent phase b. Protracted active phase c. Arrest of active phase d. Protracted descent

ANS: C With an arrest of the active phase, the progress of labor has stopped. This patient has not had any anticipated cervical change, thus indicating an arrest of labor. In the nulliparous woman a prolonged latent phase typically would last more than 20 hours. A protracted active phase, the first or second stage of labor, would be prolonged (slow dilation). With protracted descent, the fetus would fail to descend at an anticipated rate during the deceleration phase and second stage of labor

A primigravida at 40 weeks of gestation is having uterine contractions every 1.5 to 2 minutes and says that they are very painful. Her cervix is dilated 2 cm and has not changed in 3 hours. The woman is crying and wants an epidural. What is the likely status of this woman's labor? a. She is exhibiting hypotonic uterine dysfunction. b. She is experiencing a normal latent stage. c. She is exhibiting hypertonic uterine dysfunction. d. She is experiencing pelvic dystocia.

ANS: C Women who experience hypertonic uterine dysfunction, or primary dysfunctional labor, often are anxious first-time mothers who are having painful and frequent contractions that are ineffective at causing cervical dilation or effacement to progress. With hypotonic uterine dysfunction, the woman initially makes normal progress into the active stage of labor; then the contractions become weak and inefficient or stop altogether. The contraction pattern seen inthis woman signifies hypertonic uterine activity. Typically, uterine activity in this phase occurs at 4- to 5-minute intervals lasting 30 to 45 seconds. Pelvic dystocia can occur whenever contractures of the pelvic diameters reduce the capacity of the bony pelvis, including the inlet, mid-pelvis, outlet, or any combination of these planes.

The baseline fetal heart rate (FHR) is the average rate during a 10-minute segment. Changes in FHR are categorized as periodic or episodic. These patterns include both accelerations and decelerations. The labor nurse is evaluating the patient's most recent 10-minute segment on the monitor strip and notes a late deceleration. This is likely to be caused by which physiologic alteration? (Select all that apply.) a. Spontaneous fetal movement b. Compression of the fetal head c. Placental abruption d. Cord around the baby's neck e. Maternal supine hypotension

ANS: C, E Late decelerations are almost always caused by uteroplacental insufficiency. Insufficiency is caused by uterine tachysystole, maternal hypotension, epidural or spinal anesthesia, IUGR, intraamniotic infection, or placental abruption. Spontaneous fetal movement, vaginal examination, fetal scalp stimulation, fetal reaction to external sounds, uterine contractions, fundal pressure and abdominal palpation are all likely to cause accelerations of the FHR. Early decelerations are most often the result of fetal head compression and may be caused by uterine contractions, fundal pressure, vaginal examination, and placement of an internal electrode. A variable deceleration is likely caused by umbilical cord compression. This may happen when the umbilical cord is around the baby's neck, arm, leg, or other body part or when there is a short cord, a knot in the cord, or a prolapsed cord.

In the current practice of childbirth preparation, emphasis is placed on a. the Dick-Read (natural) childbirth method. b. the Lamaze (psychoprophylactic) method. c. the Bradley (husband-coached) method. d. having expectant parents attend childbirth preparation in any or no specific method

ANS: D Encouraging expectant parents to attend childbirth preparation class is most important because preparation increases a woman's confidence and thus her ability to cope with labor and birth. Although still popular, the ―method‖ format of classes is being replaced with other offerings such as Hypnobirthing and Birthing from Within.

Which assessment is least likely to be associated with a breech presentation? a. Meconium-stained amniotic fluid b. Fetal heart tones heard at or above the maternal umbilicus c. Preterm labor and birth d. Postterm gestation

ANS: D Postterm gestation is not likely to be seen with a breech presentation. The presence of meconium in a breech presentation may result from pressure on the fetal wall as it traverses the birth canal. Fetal heart tones heard at the level of the umbilical level of the mother are a typical finding in a breech presentation because the fetal back would be located in the upper abdominal area. Breech presentations often occur in preterm births.

Which basic type of pelvis includes the correct description and percentage of occurrence in women? a. Gynecoid: classic female; heart shaped; 75% b. Android: resembling the male; wider oval; 15% c. Anthropoid: resembling the ape; narrower; 10% d. Platypelloid: flattened, wide, shallow; 3%

ANS: D A platypelloid pelvis is flattened, wide, and shallow; about 3% of women have this shape. The gynecoid shape is the classical female shape, slightly ovoid and rounded; about 50% of women have this shape. An android, or male-like, pelvis is heart shaped; about 23% of women have this shape. An anthropoid, or ape-like, pelvis is oval and wider; about 24% of women have this shape.

The most common cause of decreased variability in the fetal heart rate (FHR) that lasts 30 minutes or less is a. altered cerebral blood flow. b. fetal hypoxemia. c. umbilical cord compression. d. fetal sleep cycles

ANS: D A temporary decrease in variability can occur when the fetus is in a sleep state. These sleep states do not usually last longer than 30 minutes. Altered fetal cerebral blood flow would result in early decelerations in the FHR. Fetal hypoxemia would be evidenced by tachycardia initially and then bradycardia. A persistent decrease or loss of FHR variability may be seen. Umbilical cord compression would result in variable decelerations in the FHR

The obstetric nurse is preparing the patient for an emergency cesarean birth, with no time to administer spinal anesthesia. The nurse is aware and prepared for the greatest risk of administering general anesthesia to the patient. This risk is a. respiratory depression. b. uterine relaxation. c. inadequate muscle relaxation. d. aspiration of stomach contents.

ANS: D Aspiration of acidic gastric contents with possible airway obstruction is a potentially fatal complication of general anesthesia. Respirations can be altered during general anesthesia, and the anesthesiologist will take precautions to maintain proper oxygenation. Uterine relaxation can occur with some anesthesia; however, this can be monitored and prevented. Inadequate muscle relaxation can be improved with medication.

A woman at 26 weeks of gestation is being assessed to determine whether she is experiencing preterm labor. What finding indicates that preterm labor is occurring? a. Estriol is not found in maternal saliva. b. Irregular, mild uterine contractions are occurring every 12 to 15 minutes. c. Fetal fibronectin is present in vaginal secretions. d. The cervix is effacing and dilated to 2 cm

ANS: D Cervical changes such as shortened endocervical length, effacement, and dilation are predictors of imminent preterm labor. Changes in the cervix accompanied by regular contractions indicate labor at any gestation. Estriol is a form of estrogen produced by the fetus that is present in plasma at 9 weeks of gestation. Levels of salivary estriol have been shown to increase before preterm birth. Irregular, mild contractions that do not cause cervical change are not considered a threat. The presence of fetal fibronectin in vaginal secretions between 24 and 36 weeks of gestation could predict preterm labor, but it has only a 20% to 40% positive predictive value. Of more importance are other physiologic clues of preterm labor such as cervical changes.

An 18-year-old pregnant woman, gravida 1, is admitted to the labor and birth unit with moderate contractions every 5 minutes that last 40 seconds. The woman states, ―My contractions are so strong that I don't know what to do with myself.‖ The nurse should a. assess for fetal well-being. b. encourage the woman to lie on her side. c. disturb the woman as little as possible. d. recognize that pain is personalized for each individual

ANS: D Each woman's pain during childbirth is unique and is influenced by a variety of physiologic, psychosocial, and environmental factors. A critical issue for the nurse is how support can make a difference in the pain of the woman during labor and birth. Assessing for fetal well-being includes no information that would indicate fetal distress or a logical reason to be overly concerned about the well-being of the fetus. The left lateral position is used to alleviate fetal distress, not maternal stress. The nurse has an obligation to provide physical, emotional, and psychosocial care and support to the laboring woman. This patient clearly needs support

In order to care for obstetric patients adequately, the nurse understands that labor contractions facilitate cervical dilation by a. contracting the lower uterine segment. b. enlarging the internal size of the uterus. c. promoting blood flow to the cervix. d. pulling the cervix over the fetus and amniotic sac.

ANS: D Effective uterine contractions pull the cervix upward at the same time that the fetus and amniotic sac are pushed downward. The contractions are stronger at the fundus. The internal size becomes smaller with the contractions; this helps to push the fetus down. Blood flow decreases to the uterus during a contraction.

When assessing the relative advantages and disadvantages of internal and external electronic fetal monitoring, nurses comprehend that both a. can be used when membranes are intact. b. measure the frequency, duration, and intensity of uterine contractions. c. may need to rely on the woman to indicate when uterine activity (UA) is occurring. d. can be used during the antepartum and intrapartum periods.

ANS: D External monitoring can be used in both periods; internal monitoring can be used only in the intrapartum period. For internal monitoring the membranes must have ruptured, and the cervix must be sufficiently dilated. Internal monitoring measures the intensity of contractions; external monitoring cannot do this. With external monitoring, the woman may need to alert the nurse that UA is occurring; internal monitoring does not require this.

With regard to the care management of preterm labor, nurses should be aware that a. all women must be considered at risk for preterm labor and prediction is so hit-and-miss, teaching pregnant women the symptoms probably causes more harm through false alarms. b. Braxton Hicks contractions often signal the onset of preterm labor. c. preterm labor is likely to be the start of an extended labor, a woman with symptoms can wait several hours before contacting the primary caregiver. d. the diagnosis of preterm labor is based on gestational age, uterine activity, and progressive cervical change.

ANS: D Gestational age of 20 to 37 weeks, uterine contractions, and a cervix that is 80% effaced or dilated 2 cm indicates preterm labor. It is essential that nurses teach women how to detect the early symptoms of preterm labor. Braxton Hicks contractions resemble preterm labor contractions, but they are not true labor. Waiting too long to see a health care provider could result in not administering essential medications. Preterm labor is not necessarily long-term labor.

Which statement correctly describes the effects of various pain factors? a. Higher prostaglandin levels arising from dysmenorrhea can blunt the pain of childbirth. b. Upright positions in labor increase the pain factor because they cause greater fatigue. c. Women who move around trying different positions are experiencing more pain. d. Levels of pain-mitigating -endorphins are higher during a spontaneous, natural childbirth.

ANS: D Higher endorphin levels help women tolerate pain and reduce anxiety and irritability. Higher prostaglandin levels correspond to more severe labor pains. Upright positions in labor usually result in improved comfort and less pain. Moving freely to find more comfortable positions is important for reducing pain and muscle tension.

Nurses should be aware that the induction of labor a. can be achieved by external and internal version techniques. b. is also known as a trial of labor (TOL). c. is almost always done for medical reasons. d. is rated for viability by a Bishop score

ANS: D Induction of labor is likely to be more successful with a Bishop score of 9 or higher for first-time mothers and 5 or higher for veterans. Version is turning of the fetus to a better position by a physician for an easier or safer birth. A trial of labor is the observance of a woman and her fetus for several hours of active labor to assess the safety of vaginal birth. Two thirds of cases of induced labor are elective and are not done for medical reasons.

The standard of care for obstetrics dictates that an internal version may be used to manipulate the a. fetus from a breech to a cephalic presentation before labor begins. b. fetus from a transverse lie to a longitudinal lie before cesarean birth. c. second twin from an oblique lie to a transverse lie before labor begins. d. second twin from a transverse lie to a breech presentation during vaginal birth.

ANS: D Internal version is used only during vaginal birth to manipulate the second twin into a presentation that allows it to be born vaginally. For internal version to occur, the cervix needs to be completely dilated.

For the labor nurse, care of the expectant mother begins with any or all of these situations, with the exception of a. the onset of progressive, regular contractions. b. the bloody, or pink, show. c. the spontaneous rupture of membranes. d. formulation of the woman's plan of care for labor.

ANS: D Labor care begins when progressive, regular contractions begin; the blood-tinged mucoid vaginal discharge appears; or fluid is discharged from the vagina. The woman and nurse can formulate their plan of care before labor or during treatment.

Which fetal heart rate (FHR) finding would concern the nurse during labor? a. Accelerations with fetal movement b. Early decelerations c. An average FHR of 126 beats/min d. Late decelerations

ANS: D Late decelerations are caused by uteroplacental insufficiency and are associated with fetal hypoxemia. They are considered ominous if persistent and uncorrected. Accelerations in the FHR are an indication of fetal well-being. Early decelerations in the FHR are associated with head compression as the fetus descends into the maternal pelvic outlet; they generally are not a concern during normal labor.

When using intermittent auscultation (IA) for fetal heart rate, nurses should be aware that a. they can be expected to cover only two or three patients when IA is the primary method of fetal assessment. b. the best course is to use the descriptive terms associated with electronic fetal monitoring (EFM) when documenting results. c. if the heartbeat cannot be found immediately, a shift must be made to EFM. d. ultrasound can be used to find the fetal heartbeat and reassure the mother if initial difficulty was a factor.

ANS: D Locating fetal heartbeats often takes time. Mothers can be reassured verbally and by the ultrasound pictures if ultrasound is used to help locate the heartbeat. When used as the primary method of fetal assessment, auscultation requires a nurse-to-patient ratio of one to one. Documentation should use only terms that can be numerically defined; the usual visual descriptions of EFM are inappropriate

Which method of pain management is safest for a gravida 3 para 2 admitted at 8 cm cervical dilation? a. Epidural anesthesia b. Narcotics c. Spinal block d. Breathing and relaxation techniques

ANS: D Nonpharmacologic methods of pain management may be the best option for a woman in advanced labor. It is unlikely that enough time remains to administer epidural or spinal anesthesia. A narcotic given at this time may reach its peak about the time of birth and result in respiratory depression in the newborn

As a perinatal nurse you realize that a fetal heart rate that is tachycardic, is bradycardic, or has late decelerations or loss of variability is nonreassuring and is associated with a. hypotension. b. cord compression. c. maternal drug use. d. hypoxemia.

ANS: D Nonreassuring heart rate patterns are associated with fetal hypoxemia. Fetal bradycardia may be associated with maternal hypotension. Fetal variable decelerations are associated with cord compression. Maternal drug use is associated with fetal tachycardia.

Which collection of risk factors most likely would result in damaging lacerations (including episiotomies)? a. A dark-skinned woman who has had more than one pregnancy, who is going through prolonged second-stage labor, and who is attended by a midwife. b. A reddish-haired mother of two who is going through a breech birth. c. A dark-skinned, first-time mother who is going through a long labor. d. A first-time mother with reddish hair whose rapid labor was overseen by an obstetrician.

ANS: D Reddish-haired women have tissue that is less distensible than that of darker-skinned women and therefore may have less efficient healing. First time mothers are also more at risk, especially with breech births, long second-stage labors, or rapid labors in which there is insufficient time for the perineum to stretch. The rate of episiotomies is higher when obstetricians rather than midwives attend births.

In order to evaluate the condition of the patient accurately during labor, the nurse should be aware that a. the woman's blood pressure will increase during contractions and fall back to prelabor normal between contractions. b. use of the Valsalva maneuver is encouraged during the second stage of labor to relieve fetal hypoxia. c. having the woman point her toes will reduce leg cramps. d. the endogenous endorphins released during labor will raise the woman's pain threshold and produce sedation.

ANS: D The endogenous endorphins released during labor will raise the woman's pain threshold and produce sedation. In addition, physiologic anesthesia of the perineal tissues, caused by the pressure of the presenting part, decreases the mother's perception of pain. Blood pressure increases during contractions but remains somewhat elevated between them. Use of the Valsalva maneuver is discouraged during second-stage labor because of a number of unhealthy outcomes, including fetal hypoxia. Pointing the toes can cause leg cramps, as can the process of labor itself

The priority nursing intervention after an amniotomy should be to a. assess the color of the amniotic fluid. b. change the patient's gown. c. estimate the amount of amniotic fluid. d. assess the fetal heart rate

ANS: D The fetal heart rate must be assessed immediately after the rupture of the membranes to determine whether cord prolapse or compression has occurred. Secondary to FHR assessment, amniotic fluid amount, color, odor, and consistency is assessed. Dry clothing is important for patient comfort; however, it is not the top priority.

The factors that affect the process of labor and birth, known commonly as the five Ps, include all except a. passenger. b. passageway. c. powers. d. pressure.

ANS: D The five Ps are passenger (fetus and placenta), passageway (birth canal), powers (contractions), position of the mother, and psychologic response.

If a woman complains of back labor pain, the nurse could best suggest that she a. lie on her back for a while with her knees bent. b. do less walking around. c. take some deep, cleansing breaths. d. lean over a birth ball with her knees on the floor

ANS: D The hands-and-knees position, with or without the aid of a birth ball, should help with the back pain. The supine position should be discouraged. Walking generally is encouraged

A new mother asks the nurse when the ―soft spot‖ on her son's head will go away. The nurse's answer is based on the knowledge that the anterior fontanel closes after birth by months. a. 2 b. 8 c. 12 d. 18

ANS: D The larger of the two fontanels, the anterior fontanel, closes by 18 months after birth.

The nurse who performs vaginal examinations to assess a woman's progress in labor should a. perform an examination at least once every hour during the active phase of labor. b. perform the examination with the woman in the supine position. c. wear two clean gloves for each examination. d. discuss the findings with the woman and her partner

ANS: D The nurse should discuss the findings of the vaginal examination with the woman and her partner and report them to the primary care provider. A vaginal examination should be performed only when indicated by the status of the woman and her fetus. The woman should be positioned to avoid supine hypotension. The examiner should wear a sterile glove while performing a vaginal examination for a laboring woman

The nurse practicing in a labor setting knows that the woman most at risk for uterine rupture is a. a gravida 3 who has had two low-segment transverse cesarean births. b. a gravida 2 who had a low-segment vertical incision for delivery of a 10-lb infant. c. a gravida 5 who had two vaginal births and two cesarean births. d. a gravida 4 who has had all cesarean births

ANS: D The risk of uterine rupture increases for the patient who has had multiple prior births with no vaginal births. As the number of prior uterine incisions increases, so does the risk for uterine rupture. Low-segment transverse cesarean scars do not predispose the patient to uterine rupture

A nurse may be called on to stimulate the fetal scalp a. as part of fetal scalp blood sampling. b. in response to tocolysis. c. in preparation for fetal oxygen saturation monitoring. d. to elicit an acceleration in the fetal heart rate (FHR)

ANS: D The scalp can be stimulated using digital pressure during a vaginal examination. Fetal scalp blood sampling involves swabbing the scalp with disinfectant before a sample is collected. The nurse would stimulate the fetal scalp to elicit an acceleration of the FHR. Tocolysis is relaxation of the uterus. Fetal oxygen saturation monitoring involves the insertion of a sensor.

In evaluating the effectiveness of magnesium sulfate for the treatment of preterm labor, what finding would alert the nurse to possible side effects? a. Urine output of 160 mL in 4 hours b. Deep tendon reflexes 2+ and no clonus c. Respiratory rate of 16 breaths/min d. Serum magnesium level of 10 mg/dL

ANS: D The therapeutic range for magnesium sulfate management is 5 to 8 mg/dL. A serum magnesium level of 10 mg/dL could lead to signs and symptoms of magnesium toxicity, including oliguria and respiratory distress. Urine output of 160 mL in 4 hours, deep tendon reflexes 2+ with no clonus, and respiratory rate of 16 breaths/min are normal findings.

A multiparous woman has been in labor for 8 hours. Her membranes have just ruptured. The nurse's initial response would be to a. prepare the woman for imminent birth. b. notify the woman's primary health care provider. c. document the characteristics of the fluid. d. assess the fetal heart rate and pattern.

ANS: D The umbilical cord may prolapse when the membranes rupture. The fetal heart rate and pattern should be monitored closely for several minutes immediately after ROM to ascertain fetal well-being, and the findings should be documented. Rupture of membranes (ROM) may increase the intensity and frequency of the uterine contractions, but it does not indicate that birth is imminent. The nurse may notify the primary care provider after ROM occurs and fetal well-being and the response to ROM have been assessed. The nurse's priority is to assess fetal well-being. The nurse should document the characteristics of the amniotic fluid, but the initial response is to assess fetal well-being and the response to ROM

A primigravida at 39 weeks of gestation is observed for 2 hours in the intrapartum unit. The fetal heart rate has been normal. Contractions are 5 to 9 minutes apart, 20 to 30 seconds in duration, and of mild intensity. Cervical dilation is 1 to 2 cm and uneffaced (unchanged from admission). Membranes are intact. The nurse should expect the woman to be a. admitted and prepared for a cesarean birth. b. admitted for extended observation. c. discharged home with a sedative. d. discharged home to await the onset of true labor.

ANS: D This situation describes a woman with normal assessments who is probably in false labor and will probably not deliver rapidly once true labor begins. These are all indications of false labor without fetal distress. There is no indication that further assessment or cesarean birth is indicated. The patient will likely be discharged; however, there is no indication that a sedative is needed.

A woman in the active phase of the first stage of labor is using a shallow pattern of breathing, which is about twice the normal adult breathing rate. She starts to complain about feeling light-headed and dizzy and states that her fingers are tingling. The nurse shoulda. notify the woman's physician. b. tell the woman to slow the pace of her breathing. c. administer oxygen via a mask or nasal cannula. d. help her breathe into a paper bag

ANS: D This woman is experiencing the side effects of hyperventilation, which include the symptoms of lightheadedness, dizziness, tingling of the fingers, or circumoral numbness. Having the woman breathe into a paper bag held tightly around her mouth and nose may eliminate respiratory alkalosis. This enables her to rebreathe carbon dioxide and replace the bicarbonate

You are evaluating the fetal monitor tracing of your patient, who is in active labor. Suddenly you see the fetal heart rate (FHR) drop from its baseline of 125 beats/min down to 80 beats/min. You reposition the mother, provide oxygen, increase intravenous (IV) fluid, and perform a vaginal examination. The cervix has not changed. Five minutes have passed, and the fetal heart rate remains in the 80s. What additional nursing measures should you take? a. Call for staff assistance. b. Insert a Foley catheter. c. Start Pitocin. d. Notify the care provider immediately

ANS: D To relieve an FHR deceleration, the nurse can reposition the mother, increase IV fluid, and provide oxygen. If oxytocin is infusing, it should be discontinued. If the FHR does not resolve, the primary care provider should be notified immediately. Inserting a Foley catheter is an inappropriate nursing action. If the FHR were to continue in a nonreassuring pattern, a cesarean section could be warranted, which would require a Foley catheter. However, the physician must make that determination. Pitocin may place additional stress on the fetus.

Maternity nurses often have to answer questions about the many, sometimes unusual ways people have tried to make the birthing experience more comfortable. For instance, nurses should be aware that a. music supplied by the support person has to be discouraged because it could disturb others or upset the hospital routine. b. women in labor can benefit from sitting in a bathtub, but they must limit immersion to no longer than 15 minutes at a time. c. effleurage is permissible, but counterpressure is almost always counterproductive. d. electrodes attached to either side of the spine to provide high-intensity electrical impulses facilitate the release of endorphins.

ANS: D Transcutaneous electrical nerve stimulation does help. Music may be very helpful for reducing tension and certainly can be accommodated by the hospital. Women can stay in a bath as long as they want, although repeated baths with breaks may be more effective than a long soak. Counterpressure can help the woman cope with lower back pain.

The nurse expects to administer an oxytocic (e.g., Pitocin, Methergine) to a woman after expulsion of her placenta to a. relieve pain. b. stimulate uterine contraction. c. prevent infection. d. facilitate rest and relaxation

NS: B Oxytocics stimulate uterine contractions, which reduce blood loss after the third stage of labor. Oxytocics are not used to treat pain or prevent infection. They cause the uterus to contract, which reduces blood loss. Oxytocics do not facilitate rest and relaxation.

In documenting labor experiences, nurses should know that a uterine contraction is described according to all these characteristics except a. frequency (how often contractions occur). b. intensity (the strength of the contraction at its peak). c. resting tone (the tension in the uterine muscle). d. appearance (shape and height).

NS: D Uterine contractions are described in terms of frequency, intensity, duration, and resting tone

Which factors influence cervical dilation? (Select all that apply.) a. Strong uterine contractions b. The force of the presenting fetal part against the cervix c. The size of the female pelvis d. The pressure applied by the amniotic sac e. Scarring of the cervix

aNS: A, B, D, E Dilation of the cervix occurs by the drawing upward of the musculofibrous components of the cervix, which is caused by strong uterine contractions. Pressure exerted by the amniotic fluid while the membranes are intact or by the force applied by the presenting part also can promote cervical dilation. Scarring of the cervix as a result of a previous infection or surgery may slow cervical dilation. Pelvic size does not affect cervical dilation.

While evaluating an external monitor tracing of a woman in active labor whose labor is being induced, the nurse notes that the fetal heart rate (FHR) begins to decelerate at the onset of several contractions and returns to baseline before each contraction ends. The nurse should a. change the woman's position. b. discontinue the oxytocin infusion. c. insert an internal monitor. d. document the finding in the patient's record

aNS: D The FHR indicates early decelerations, which are not an ominous sign and do not require any intervention. The nurse should simply document these findings

The nurse providing care for the laboring woman should understand that amnioinfusion is used to treat a. variable decelerations. b. late decelerations. c. fetal bradycardia. d. fetal tachycardia

ans A Amnioinfusion is used during labor either to dilute meconium-stained amniotic fluid or to supplement the amount of amniotic fluid to reduce the severity of variable decelerations caused by cord compression. Amnioinfusion has no bearing on late decelerations, fetal bradycardia, or fetal tachycardia alterations in fetal heart rate (FHR) tracings.

A maternal indication for the use of forceps is a. a wide pelvic outlet. b. maternal exhaustion. c. a history of rapid deliveries. d. failure to progress past 0 station.

ans B A mother who is exhausted may be unable to assist with the expulsion of the fetus. The patient with a wide pelvic outlet will likely not require vacuum extraction. With a rapid delivery, vacuum extraction is not necessary. A station of 0 is too high for a vacuum extraction

When a nulliparous woman telephones the hospital to report that she is in labor, the nurse initially should a. tell the woman to stay home until her membranes rupture. b. emphasize that food and fluid intake should stop. c. arrange for the woman to come to the hospital for labor evaluation. d. ask the woman to describe why she believes she is in labor.

ans D Assessment begins at the first contact with the woman, whether by telephone or in person. By asking the woman to describe her signs and symptoms, the nurse can begin the assessment and gather data. The amniotic membranes may or may not spontaneously rupture during labor. The patient may be instructed to stay home until the uterine contractions become strong and regular. The nurse may want to discuss the appropriate oral intake for early labor such as light foods or clear liquids, depending on the preference of the patient or her primary health care provider. Before instructing the woman to come to the hospital, the nurse should initiate the assessment during the telephone interview

A laboring woman received an opioid agonist (meperidine) intravenously 90 minutes before she gave birth. Which medication should be available to reduce the postnatal effects of Demerol on the neonate? a. Fentanyl (Sublimaze) b. Promethazine (Phenergan) c. Naloxone (Narcan) d. Nalbuphine (Nubain)

ans c An opioid antagonist can be given to the newborn as one part of the treatment for neonatal narcosis, which is a state of central nervous system (CNS) depression in the newborn produced by an opioid. Opioid antagonists such as naloxone (Narcan) can promptly reverse the CNS depressant effects, especially respiratory depression. Fentanyl, promethazine, and nalbuphine do not act as opioid antagonists to reduce the postnatal effects of Demerol on the neonate. Although meperidine (Demerol) is a low-cost medication and readily available, the use of Demerol in labor has been controversial because of its effects on the neonate

Because the risk for childbirth complications may be revealed, nurses should know that the point of maximal intensity (PMI) of the fetal heart tone (FHT) is a. usually directly over the fetal abdomen. b. in a vertex position heard above the mother's umbilicus. c. heard lower and closer to the midline of the mother's abdomen as the fetus descends and rotates internally. d. in a breech position heard below the mother's umbilicus

ans c Nurses should be prepared for the shift. The PMI of the FHT usually is directly over the fetal back. In a vertex position it is heard below the mother's umbilicus. In a breech position it is heard above the mother's umbilicus.

Induction of labor is considered an acceptable obstetric procedure if it is in the best interest to deliver the fetus. The charge nurse in the labor and delivery unit is often asked to schedule patients for this procedure and therefore must be cognizant of the specific conditions appropriate for labor induction. These include (Select all that apply.) a. rupture of membranes at or near term. b. convenience of the woman or her physician. c. chorioamnionitis (inflammation of the amniotic sac). d. postterm pregnancy. e. fetal death.

ansa,c,d,e These are all acceptable indications for induction. Other conditions include intrauterine growth retardation (IUGR), maternal-fetal blood incompatibility, hypertension, and placental abruption. Elective inductions for the convenience of the woman or her provider are not recommended; however, they have become commonplace. Factors such as rapid labors and living a long distance from a health care facility may be valid reasons in such a circumstance. Elective delivery should not occur before 39 weeks' completed gestation

The least common cause of long, difficult, or abnormal labor (dystocia) is a. midplane contracture of the pelvis. b. compromised bearing-down efforts as a result of pain medication. c. disproportion of the pelvis. d. low-lying placenta.

ansc

While evaluating an external monitor tracing of a woman in active labor, the nurse notes that the fetal heart rate (FHR) for five sequential contractions begins to decelerate late in the contraction, with the nadir of the decelerations occurring after the peak of the contraction. The nurse's first priority is to a. change the woman's position. b. notify the care provider. c. assist with amnioinfusion. d. insert a scalp electrode.

vANS: A Late decelerations may be caused by maternal supine hypotension syndrome. They usually are corrected when the woman turns on her side to displace the weight of the gravid uterus from the vena cava. If the fetus does not respond to primary nursing interventions for late decelerations, the nurse would continue with subsequent intrauterine resuscitation measures, including notifying the care provider. An amnioinfusion may be used to relieve pressure on an umbilical cord that has not prolapsed. The FHR pattern associated with this situation most likely reveals variable deceleration. A fetal scalp electrode would provide accurate data for evaluating the well-being of the fetus; however, this is not a nursing intervention that would alleviate late decelerations, nor is it the nurse's first priority.


Related study sets

Module 2: Connecting and Communicating Online: The Internet, Websites, and Media

View Set

(APES Final Review) Chapter 6 Biomes

View Set

MGT 3900 Conceptual Final Review

View Set

Chapter 4 : Folk and Popular Culture

View Set

Chapter 2- Life Basics -A.D Banker

View Set

Chapter 07 Mediated Communication and Social Media

View Set

HESI Review Test-Maternity, HESI Exit 5, OB HESI/FINAL, HESI with Rationale 12, HESI OB/MATERNITY Practice Quiz, HESI

View Set

Chapter 11: Technology, Production, and Cost

View Set